Last min aapmr qbank review Flashcards

1
Q

Pain after FOOSH. Tenderness with pain to palpation distal to the ulnar styloid between the FCU and ECU.

Dx?
Next best step?

A

triangular fibrocartilage complex (TFCC).

This structure is a stabilizer of the distal radioulnar joint and is composed of an avascular articular disc and radioulnar ligament complex. It is often injured with repetitive wrist activities or compressive loads.

TFCC tears are best imaged by magnetic resonance imaging (MRI). Injury to this complex would not be optimally evaluated on plain films, bone scan, or computed tomography scan.

How well did you know this?
1
Not at all
2
3
4
5
Perfectly
2
Q

baseball player had a twisting valgus deviation of the right knee during play. The MRI showed edema at the inferomedial patella and at the right lateral femoral condyle with ruptured fibers at the medial patellofemoral ligament. What is the most likely diagnosis based on MRI findings?

A

According to the MRI there is edema at the inferomedial patella and lateral femoral condyle, which are classic findings of patellar dislocation. When the patella returns to its natural position (after the traumatic patellar dislocation) the inferomedial portion of the patella touches the lateral femoral condyle, causing the classic findings at the MRI.

How well did you know this?
1
Not at all
2
3
4
5
Perfectly
3
Q

Which of the following is optimal initial treatment for congenital hip dysplasia in infants less than 6 months?

A

The Pavlik harness is the first line treatment. It is a brace which fastens around an infant’s legs and connects to the shoulders and torso to prevent mal-tracking of the femoral head within the acetabulum. Spica casting requires a series of hip casting to prevent femoral acetabular dislocation. Abduction braces are generally prescribed after completion of a spica cast series. Closed reduction is not the best initial treatment.

How well did you know this?
1
Not at all
2
3
4
5
Perfectly
4
Q

How long are hip precautions routinely continued after a posterior approach total hip arthroplasty?

A

Total hip precautions include avoiding flexion of more than 90 degrees, adduction past midline, and internal rotation past neutral. Total hip precautions for a posterior approach total hip arthroplasty should be continued for 12 weeks.

How well did you know this?
1
Not at all
2
3
4
5
Perfectly
5
Q

Treatment/management for stingers?

A

Stingers are a transient episode of unilateral pain and/or paresthesias in an upper extremity. There is a preponderance of C5 or C6 and upper trunk symptoms. All athletes require further diagnostic evaluation, with the exception of a first stinger (or second stinger in a separate season) with rapid resolution of symptoms prior to return to play. Athletes must have resolution of all symptoms with full, pain-free cervical ROM and full strength, along with an absence of any underlying risk factors for further injury, before they are allowed to return to play

How well did you know this?
1
Not at all
2
3
4
5
Perfectly
6
Q

Risk factors for plantar fasciitis

A

Obesity is a risk factor for plantar fasciitis. Plantar fasciitis affects both men and women equally, most commonly between the ages of 40 and 70 years. Factors that increase the tension on the plantar fascia, such as decreased subtalar motion, pes cavus, pes planus, and a tight Achilles’ tendon, may contribute to plantar fasciitis.

How well did you know this?
1
Not at all
2
3
4
5
Perfectly
7
Q

A 4-year-old child suffers an ankle inversion. When you examine him the following day, he has mild swelling laterally and is tender to palpation approximately 2-3 cm proximal to the distal aspect of the lateral malleolus and he reacts to pinch. The remainder of the examination is limited due to pain. Radiographs appear normal. What is the management of this patient?

A

Short leg non-weight bearing cast for 3 weeks.

Given the mechanism of injury, location of pain, and “normal” radiographs, the boy likely has a Salter Harris I Fracture of the distal fibular physis. A Salter Harris I fracture is a transverse fracture through the hypertrophic zone of the physis, and is typically not visible on radiographs though there may be widening of the growth plate. A walking boot is not sufficient to immobilize the ankle joint in a toddler. A CT scan is not required given there is not any significant abnormality on radiographs. A MRI to rule out an ATFL tear is not indicated. There is a significantly greater likelihood of a bony versus ligamentous injury.

How well did you know this?
1
Not at all
2
3
4
5
Perfectly
8
Q

A 4-year-old child suffers an ankle inversion. When you examine him the following day, he has mild swelling laterally and is tender to palpation approximately 2-3 cm proximal to the distal aspect of the lateral malleolus and he reacts to pinch. The remainder of the examination is limited due to pain. Radiographs appear normal. What is the management of this patient?

A

Short leg non-weight bearing cast for 3 weeks.

Given the mechanism of injury, location of pain, and “normal” radiographs, the boy likely has a Salter Harris I Fracture of the distal fibular physis. A Salter Harris I fracture is a transverse fracture through the hypertrophic zone of the physis, and is typically not visible on radiographs though there may be widening of the growth plate. A walking boot is not sufficient to immobilize the ankle joint in a toddler. A CT scan is not required given there is not any significant abnormality on radiographs. A MRI to rule out an ATFL tear is not indicated. There is a significantly greater likelihood of a bony versus ligamentous injury.

How well did you know this?
1
Not at all
2
3
4
5
Perfectly
9
Q

An adult patient is referred with a diagnosis of generalized hypermobility syndrome and a Beighton score of 8 points.

What is the best advice?

A

The Beighton score is a 9 point scale and a score of 5 or 6 is consistent with a hypermobility diagnosis. Family history is commonly positive but in the absence of hyperelastic skin, vascular complications, or neurologic signs, suggesting neuromuscular disease genetic testing is generally negative for hypermobile EDS. Pain and fatigue are common and may be severe and disabling; extensor, core strengthening and endurance training are indicated to improve symptoms and function.

How well did you know this?
1
Not at all
2
3
4
5
Perfectly
10
Q

What is the normal bladder capacity for a two-year old?

A

The bladder capacity in children is based on the Berger equation –age in years plus 2 – equals bladder capacity in ounces up until around age 12 years when the bladder capacity equals an adult size. If the detrusor muscle is overactive and can overcome the pressure of the external sphincter, the result will likely be a small capacity bladder. If the external sphincter is open, there will also likely have a small capacity bladder unless there is have a large flaccid bladder.

How well did you know this?
1
Not at all
2
3
4
5
Perfectly
11
Q

A 13-year-old right hand dominant baseball pitcher is seen in your clinic with right lateral shoulder pain that is aggravated by throwing. He denies any prior shoulder injury. Your examination is significant only for tenderness along the lateral deltoid. Radiographs of right shoulder are normal. The next best step in management of this patient is:

A

XR L shoulder

This adolescent has the hallmark history and potential signs of Little League Shoulder (LLS)/Glenohumeral Epiphysiolysis. LLS is a repetitive stress injury due to overuse and improper rest that typically occurs in young teenagers. It is likely a result of torsional overload of the proximal humeral epiphysis during maximal shoulder external rotation during the throwing cycle. In order to fully compare the physes, contra-lateral radiographs are needed as even subtle differences can make the diagnosis. MRI imaging not as efficient and more expensive. Treatment recommendations include elimination of throwing for 6 weeks after diagnosis and an additional 6 weeks of rehabilitation without throwing. A return to throw/pitch program is initiated once the patient is pain free and has completed a rehabilitation program without setbacks.

How well did you know this?
1
Not at all
2
3
4
5
Perfectly
12
Q

A 20-year-old college recreational athlete presents to your clinic after injuring his right ankle during a flag football game earlier in the day. On exam you note an antalgic gait with significant swelling, bruising, and tenderness to palpation at the anterior aspect of the lateral malleolus. No tenderness along the lateral and posterior aspect of the lateral malleolus, the medial malleolar region, or the foot. Regarding imaging, what should you order?

A

Given the history and examination, this patient most likely has a lateral ankle sprain. The Ottawa ankle and foot rules offer a high degree of sensitivity for fractures of the foot and ankle in patients presenting with acute ankle or foot pain. While the patient has pain in the lateral malleolar zone, he is able to bear weight, has no foot pain, and his ankle pain is not located at the posterior aspect of the distal 6cm of the fibula; hence, he does not satisfy criteria for obtaining an x-ray (with stress views or otherwise). There is no indication for ordering an MRI at this time.

How well did you know this?
1
Not at all
2
3
4
5
Perfectly
13
Q

In patients with degenerative disc disease of the lumbar spine, what percentage of weight do the zygapophyseal joints bear?

A

Normal zygapophyeal joints weight bear 33% of the total compressive load. Those with just zygapophyseal arthritis bear up to 47% and those with degenerative disc disease bear up to 70%.

How well did you know this?
1
Not at all
2
3
4
5
Perfectly
14
Q

Dupuytren contracture is associated with

A

Dupuytren contracture is caused by thickening and shortening of the palmar fascia and results in a flexion contracture of the MCP and PIP joints. It is associated with conditions that include alcoholism, cigarette smoking, human immunodeficiency virus infection, and diabetes. There are conflicting reports of an association with epilepsy.

How well did you know this?
1
Not at all
2
3
4
5
Perfectly
15
Q

A patient presents with pain in his ring finger after an opponent tried to slap a basketball out of his hand. On exam flexion and extension of his DIP is intact. X-ray of his finger shows a non-displaced distal phalangeal fracture. What is the next step in his treatment?

A

The goal of treatment for any finger injury is to restore the normal function of the finger. Restoration of bony anatomy is the basis for returning normal function; however, an anatomic reduction is not always necessary to achieve this goal, especially if it comes at the cost of soft tissue scarring and loss of motion. Early motion prevents adhesions of the gliding soft tissues of the extensor and flexor tendon systems and prevents contracture of the joint capsules. Immobilization of fingers much beyond 4 weeks will lead to long-term stiffness due to extensor tendon and joint capsular scarring. For example, non-articular phalangeal fractures treated with closed reduction and splinting are mobilized after 3–4 weeks, once the fractured phalanx is less tender. Even if splinting of one joint is needed, splints should be made small enough to allow early motion of uninjured joints. Therefore, in a case of a non-displaced distal phalanx fracture splinting that immobilizes the DIP joint only is sufficient. Closed non-displaced or minimally displaced fractures with acceptable alignment that are the result of a low-energy trauma usually have sufficient supporting tissues remaining intact making them stable and amenable to treatment by protected mobilization, either with local splinting of the fracture or buddy taping to adjacent fingers. Fractures with rotational or angular misalignment may be amenable to closed reduction and splinting, but these fractures are at risk for incomplete reduction and recurrent deformity. These more unstable fractures require careful and frequent clinical and radiographic follow-up. Surgical treatment is indicated for any fractures of the articular surface, open fractures, fractures with significant shortening or malrotation, and fractures which fail closed reduction. Delayed treatment of these surgically indicated fractures is always more difficult, with worse functional outcomes due to stiffness, incomplete deformity correction, and post-traumatic arthritis.

How well did you know this?
1
Not at all
2
3
4
5
Perfectly
16
Q

The most common organism responsible for septic arthritis in a one-year-old is:

A

Overall, the most common organism responsible for septic arthritis is Staphylococcus aureus. However, in children ages 2 months to 2 years old, the most common organism is Hemophilus influenza. Neisseria gonorrhea is the most common agent for septic arthritis in sexually active adolescents.

How well did you know this?
1
Not at all
2
3
4
5
Perfectly
17
Q

A 40-year-old patient is consulting you for management of her fibromyalgia and has failed to improve in the past with SSRIs, SNRIs, TCAs and Gabapentin. She asks you about other medications that have been developed for treatment of his condition. Which of the following drugs currently has the strongest research backed evidence for treatment of fibromyalgia?

A

For the pharmacological management of fibromyalgia, numerous research studies have been conducted on various drug classes. Most recently, both Tizanidine and low dose Naltrexon have been found effective only in small random controlled trials. Tramadol is classified as evidence level 5 (expert opinion) and like other opioids are generally less recommended for treating chronic pain. Cannabinoids fall under evidence level 1A (systemic review of random controlled trials) and would be the best choice among the choices above. For the pharmacological management of fibromyalgia, numerous research studies have been conducted on various drug classes. Most recently, both Tizanidine and low dose Naltrexon have been found effective only in small random controlled trials. Tramadol is classified as evidence level 5 (expert opinion) and like other opioids are generally less recommended for treating chronic pain. Cannabinoids fall under evidence level 1A (systemic review of random controlled trials) and would be the best choice among the choices above.

Gabapnetinoids (e.g. Gabapentin), TCAs (e.g. Nortryptiline) and SNRIs (e.g. Duloxetine) also all fall under evidence level 1A. In this question, the patient had not responded to these drug classes. Hence, choice D would be the best answer.

How well did you know this?
1
Not at all
2
3
4
5
Perfectly
18
Q

A 39-year-old male presents to your office after playing basketball with the inability to actively extend the DIP joint of his fourth finger. X-ray was negative for fracture. What would be your next step in the treatment of this patient?

A

A mallet finger results from the disruption of the terminal extensor tendon at its insertion on the distal phalanx that occurs most often due to impact of the fingertip on a ball or other object resulting in flexion force to the DIP joint. This is a common injury in basketball, baseball and football wide receivers. The patient will usually present with the inability to extend the DIP actively. Treatment for these injuries is a splinted DIP and an extension (PIP can be free) for 6 weeks. Surgery is only considered for fracture subluxation injuries so therefore there is no need for a surgical referral. Immobilizing with a cast is also not necessary as only the DIP needs to be immobilized. A CT scan is also not necessary as no fracture was seen on x-ray and treatment with splinting is still the treatment of choice.

How well did you know this?
1
Not at all
2
3
4
5
Perfectly
19
Q

This is performed by having the patient abduct the ipsilateral shoulder and resting the hand on top of the head. Individuals with cervical radicular symptoms will feel relief with this maneuver.

A

Bakody Sign

How well did you know this?
1
Not at all
2
3
4
5
Perfectly
20
Q

Iliotibial Band Syndrome (ITBS) is associated with which of the following rotary motions?

A

Tibial internal rotation:

ITBS has been associated with greater hip adduction and greater knee internal rotation. The concept of the knee internally rotating during flexion has implications for the biomechanics of Iliotibial Band Syndrome (ITBS). Consequently, ITBS has been associated with biomechanical abnormalities in the coronal plane, particularly at the hip, which controls orientation of the lower limb during stance. In a study of female runners, ITBS has been associated with greater peak hip adduction, and with greater peak knee internal rotation angle. Foot and ankle mechanics have not yet been shown to contribute to ITBS.

How well did you know this?
1
Not at all
2
3
4
5
Perfectly
21
Q

What is the most common complication after burn injury?

A

Hypertrophic scarring is the most common complication after burn injury, with a prevalence of 67%. Hypertrophic scars are raised, red, painful, pruritic, and contractile and stay within the margins of the original injury. Keloid scars have some of the same characteristics as hypertrophic scars, but they also extend beyond the original injury and invade into local soft tissues. Hypertrophic burn scars tend to develop in the first few months of injury, while increasing in volume and erythema. After several months, they can regress, becoming less erythematous and flatter, but the skin never returns to its original state. Younger individuals, particularly adolescents, and those with darker skin pigmentation tend to have a higher incidence of hypertrophic scarring. Wounds with a prolonged inflammatory wound healing phase and those that are open longer than 3 weeks are more likely to develop hypertrophic scars.

How well did you know this?
1
Not at all
2
3
4
5
Perfectly
22
Q

The most common bone for an osteoporotic fracture in women over age 65 is

A

Vertebral fractures are the most common osteoporotic fracture worldwide, they occur in 30-50% of people over age 50 but majority of them are asymtomatic with approximately two-thirds to three- fourths of vertebral fractures being clinically silent and often being diagnosed as an incidental finding on x-ray and less than 10% require hospital admission. Osteoporotic Hip fractures are the most common cause of hospitalization and substantially increase the risk of death and major morbidity in the elderly. The risk of hip fracture rises exponentially with age.

How well did you know this?
1
Not at all
2
3
4
5
Perfectly
23
Q

Which treatments may best prevent progression and improve symptoms of Complex Regional Pain Syndrome?

A

Range of motion exercises and NSAIDs

Early diagnosis and intervention in CRPS is associated with improved outcome and function. Rehabilitative therapies coupled with pharmacotherapy are the mainstays of early treatment. Although pain may decrease over time, detrimental changes arise from neuroplasticity. Interventional treatments are considered if conservative strategies fail. There are no well-accepted treatment guidelines for pharmacotherapy. Best evidence supports multidisciplinary care.

Medications trialed specifically for Complex Regional Pain Syndrome (CRPS) include calcitonin and bisphosphonates, corticosteroids, and most recently, intravenous immunoglobulin (IVIG). CRPS Treatments better studied in other related neuralgias include: augmentation of mono-amnergic transmission (serotonin- norepinephrine reuptake inhibitors and tricyclic anti-depressants (TCA). anti-Inflammatory Drugs/lmmunomodulators (NSAIDs, oral corticosteroids); alpha 2-adrenergic agonist (clonidine); NMDA receptor antagonists (e.g., MK-801, ketamine, amantadine, and dextromethorphan); calcium channel blockers (gabapentin, carbamazepine); and, opioids. Monotherapy is best to minimize adverse effects, cost, and patient noncompliance, but rational polypharmacy is often necessary, particularly to address different CRPS symptoms.

How well did you know this?
1
Not at all
2
3
4
5
Perfectly
24
Q

Panner’s disease?

A

Panner’s disease is bone growth disorder (osteochondrosis) of the humeral capitellum ossification centre, at the lateral aspect of the elbow[1].

How well did you know this?
1
Not at all
2
3
4
5
Perfectly
25
Q

Sever’s Disease

A

calcaneal; achilles tendon apophysitis

How well did you know this?
1
Not at all
2
3
4
5
Perfectly
26
Q

poor prognostic indicator for seronegative spondyloarthropathies?

A

Poor prognostic indicators include a younger age of onset, greater peripheral than axial joint involvement, uveitis, and elevated erythrocyte sedimentation rate (ESR), and poor response to non-steroidal anti-inflammatory drugs (NSAIDs).

How well did you know this?
1
Not at all
2
3
4
5
Perfectly
27
Q

How long is an abduction pillow utilized after a posterior approach hip replacement?

A

An abduction pillow prevents posterior dislocation of the hip prosthesis when the patient is lying in bed and should be used for approximately 6-12 weeks.

How well did you know this?
1
Not at all
2
3
4
5
Perfectly
28
Q

What percent of rheumatoid arthritis patients are rheumatoid factor positive?

A

70-80% of rheumatoid arthritis patients are RF (+), which carries a poorer prognosis.

How well did you know this?
1
Not at all
2
3
4
5
Perfectly
29
Q

The American Academy of Orthopedic Surgeons recommends weight loss to patients with symptomatic knee arthritis as a strategy to manage symptoms and improve function when the body mass index is calculated to be greater than or equal to ____ kg/m2:

A

The American Academy of Orthopedic Surgeons practice guidelines recommend advising patients with a body mass index greater/equal to 25 to use weight loss as a strategy to help with pain reduction and improvement in function.

How well did you know this?
1
Not at all
2
3
4
5
Perfectly
30
Q

A 22-year-old female runner presents to your clinic for right hip pain. She has been training hard for an upcoming half marathon. Recent MRI indicates a stress fracture at the superior aspect of the femoral neck. There is no clear cortical break. Based on these findings, what is the best next step in management?

A

Femoral neck stress fractures are categorized as “critical” due to the tenuous vascularity and the potential for poor outcomes with progression to fracture completion and displacement. Stress fractures of the inferior aspect may be managed non-operatively as this is the side of the femoral neck that is loaded in compression, which is relatively favorable for bone loading (exception: cortical breaks should be referred for evaluation). Management of inferior aspect, or “compression sided” stress fractures involves a period of 6-8 weeks of non-weight bearing followed by a slow progressive return to weight bearing, followed by physical therapy, addressing training and biomechanical errors, and ending with a slow progressive return to run program. However, stress fractures of the superior aspect of the femoral neck are particularly high-risk for completion and displacement, as the bone at the superior aspect is being loaded in tension. Non weight bearing status and urgent orthopedic surgical consultation is recommended.

How well did you know this?
1
Not at all
2
3
4
5
Perfectly
31
Q

A 27-year-old golfer presents with pain on the volar ulnar aspect of his hand. X-ray shows he has a fracture of the hook of his hamate bone. What is the treatment of choice?

A

Hamate hook fractures represent 2-4% of all carpal fractures and are a common injury in golf, baseball and hockey. Patients usually present with vague complaints of pain at the volar ulnar aspect of their hand with pain provoked when attempting a tight grip. Plain radiographs may not visualize the fracture thus a CT scan should be considered if one has a high suspicion for fracture. Excision of the hook of the hamate is considered the treatment of choice. Acute injuries and non-displaced fractures may be treated non-operatively but excision is the treatment of choice. Nonunion rates greater than 50% and as high as 80-90% can occur with conservative treatment. Therefore, all hamate hook fractures should be referred to a hand surgeon for possible surgical intervention.

How well did you know this?
1
Not at all
2
3
4
5
Perfectly
32
Q

What condition is commonly associated with lateral epicondylitis?

A

Lateral epicondylitis, or proximal wrist extensor tendinopathy, is associated with smoking and obesity. It occurs equally among males and females. It is not associated with hypercholesterolemia

How well did you know this?
1
Not at all
2
3
4
5
Perfectly
33
Q

What percentage of low back pain has a specific identifiable cause?

A

5-15%

How well did you know this?
1
Not at all
2
3
4
5
Perfectly
34
Q

Seat belt use during motor vehicle collisions has not reduced incidence of spinal injuries associated with

A

Thoracic and lumbar spine fracture patterns are influenced by the age of occupant and type and use of seat belts. Despite a reduction in overall injury severity and mortality, seat belt use is associated with an increased incidence of thoracic and lumbar spine fractures. Minor thoracic and lumbar spine fractures were associated with an increased likelihood of pelvic and abdominal injuries and higher Injury Severity Scores (ISS), demonstrating their importance in predicting overall injury severity. Extension injuries occurred in older obese individuals and were associated with a high fatality rate. Future advancements in automobile safety engineering should address the need to reduce thoracic and lumbar spine injuries in belted occupants.

How well did you know this?
1
Not at all
2
3
4
5
Perfectly
35
Q

A 15-year-old high school football player sustains a direct hit to the shoulder. He feels immense shoulder pain and decreased range of motion with associated numbness in his hand. Sideline evaluation confirms a glenohumeral joint dislocation and neurological symptoms resolve after joint reduction. Post-reduction radiographs confirm reduction and reveal a sizeable osseous depression in the posterior-lateral aspect of the humeral head. After providing a sling for comfort, what is the next best step in evaluation?

A

Young, active athletes with a first-time shoulder dislocation have a significant likelihood of suffering another dislocation in the future. Recurrence risk factors include age, activity level, in vs out of season, sex (male), and prior history of instability events with bony defects and/or capsular or ligamentous injuries. A true glenohumeral dislocation with an associated Hill Sachs lesion is an indication of instability of the shoulder joint. Absolute indications for an early orthopaedic consultation include a humeral head articular surface osseous defect of greater than 25%, greater than 50% rotator cuff tear, glenoid osseous defect greater than 25%, humeral head articular surface osseous defect greater than 25%, proximal humerus fracture requiring surgery, irreducible dislocation, interposed tissue or nonconcentric reduction, failed trial of rehabilitation, inability to tolerate shoulder restrictions, or inability to perform sport-specific drills without instability. There is no indication for an EMG given resolution of neurological symptoms after reduction. Sling use is limited to 1-3 weeks. Typically, physical therapy is not started within the first 1-2 weeks due to risk of re-dislocation.

How well did you know this?
1
Not at all
2
3
4
5
Perfectly
36
Q

Which are factors that increase the risk of developing hypertrophic scars after burn injury?

A

Hypertrophic scars are more prevalent in larger and deeper burns, in younger patients, those with darker skin, across areas of motion like joints, and when wounds are left open for prolonged periods of time (over 3 weeks).

How well did you know this?
1
Not at all
2
3
4
5
Perfectly
37
Q

A C6/C7 disc herniation into the neural foramen is most likely to produce paresthesias in which of the following areas?

A

Cervical disc herniations into the neural foramen affect the exiting nerve root, which is named by the lower vertebral body of the foramen. This nomenclature switches to the upper vertebral body after the emergence of the C8 nerve root between the C7 and T1 vertebrae. Thus, a C6/C7 foraminal disc herniation would compromise the C7 nerve root. The C7 nerve root dermatome is mapped to the third digit. The thumb represents the C6 dermatome, the fifth digit represents the C8 dermatome, and the medial epicondyle represents the T1 dermatome.

How well did you know this?
1
Not at all
2
3
4
5
Perfectly
38
Q

n young athletes, the likelihood of a 2nd shoulder dislocation after experiencing a first-time dislocation is:

A

Recurrence rates of shoulder dislocations in young athletes (<30 years old) range from approximately 60-90%. The t high re-dislocation rate is due to the fact that younger patients with strong, healthy rotator cuff tissue can withstand a high-energy insult but their weaker anterior static restraints (ie, labrum, shoulder capsule) cannot. Bracing may reduce the risk of recurrence, but restricts motion and may not be tolerated in certain sport-specific tasks such as throwing.

How well did you know this?
1
Not at all
2
3
4
5
Perfectly
38
Q

n young athletes, the likelihood of a 2nd shoulder dislocation after experiencing a first-time dislocation is:

A

Recurrence rates of shoulder dislocations in young athletes (<30 years old) range from approximately 60-90%. The t high re-dislocation rate is due to the fact that younger patients with strong, healthy rotator cuff tissue can withstand a high-energy insult but their weaker anterior static restraints (ie, labrum, shoulder capsule) cannot. Bracing may reduce the risk of recurrence, but restricts motion and may not be tolerated in certain sport-specific tasks such as throwing.

How well did you know this?
1
Not at all
2
3
4
5
Perfectly
39
Q

The HLA-B27 spondyloarthropathies include

A

psoriatic arthritis, ankylosing spondylitis, oligoarticular JIA, enteropathic arthropathy, and reactive arthritis (formerly known as Reiter syndrome).

How well did you know this?
1
Not at all
2
3
4
5
Perfectly
40
Q

What percentage of fractures in people older than 45 are related to osteoporosis?

A

70% of fractures in people over 45 years of age are related to osteoporosis. 1/3 of females greater than 65 years of age will have vertebral fractures. Hip fractures are the greatest cause of morbidity and mortality as related to osteoporosis.

How well did you know this?
1
Not at all
2
3
4
5
Perfectly
41
Q

Pseudogout is associated with which other conditions?

A

associated with hyperparathyroidism, hypothyroidism, hypomagnesemia, hypophosphatemia, hemochromatosis, and amyloidosis.

How well did you know this?
1
Not at all
2
3
4
5
Perfectly
42
Q

Which conditions are commonly associated with sacroiliitis?

A

Inflammatory bowel disease, reactive arthritis, psoriatic arthritis, septic arthritis, and ankylosing spondylitis are all associated with sacroiliitis. Lupus is not commonly associated with sacroiliitis.

Traditionally, there were four subtypes of spondyloarthritides: ankylosing spondylitis (AS), psoriatic arthritis (PsA), reactive arthritis (formerly Reiter Syndrome) (ReA), and spondyloarthropathy associated with inflammatory bowel disease (IBD). Recent developments have expanded the concept of SpA to include undifferentiated spondyloarthropathy (USpA). This category is for patients who are diagnosed with SpA, but do not fall into any specific subtype.

How well did you know this?
1
Not at all
2
3
4
5
Perfectly
43
Q

Jersey Finger

A

Complete or incomplete injury to the flexor tendon (superficialis and/or profundus) may be spontaneous as in the case of rheumatoid arthritis or more commonly due to a traumatic nature as seen in athletes in sports like football or wrestling. The classic mechanism of injury in athletes is when a player’s finger gets caught in the jersey of another when attempting to grab him. The profundus tendon is avulsed from its insertion and possibly accompanied by a bony fragment (usually the fourth digit). This is called Jersey Finger. The patient is unable to actively flex the DIP joint. In order to effectively test the function of the flexor digitorum profundus, the patient must flex the DIP while the examiner holds the PIP joint in extension. No imaging is really needed for diagnosis but plain films may show an avulsed fragment near the tendonous insertion. The treatment for Jersey Finger is hand surgery referral for early surgical repair.

How well did you know this?
1
Not at all
2
3
4
5
Perfectly
44
Q

A patient presents with an injury to his nail bed and a subungual hematoma that covers >50 % of the nail matrix. What is the next step in the management of this patient?

A

With any nail bed injury a plain radiograph should always be done to rule out any fracture of the phalanx. A subungual hematoma may also accumulate which can be very painful. This can be decompressed using cautery or an 18-gauge needle. If one has avulsion of the nail or a complex injury with loss of the nail bed, a surgical consult for possible repair is needed. If there is no damage to the matrix, observation and cleansing with a non-adherent dressing is recommended. Therefore, the correct and best answer in the next step managing this patient would be to x-ray the finger to rule out any type of fracture before any further treatment is necessary.

How well did you know this?
1
Not at all
2
3
4
5
Perfectly
45
Q

Indicators of poor outcome of juvenile idiopathic arthritis include

A

Indicators of poor outcome of juvenile idiopathic arthritis include greater severity or extent of arthritis at onset, late presentation to pediatric rheumatology, prolonged steroid use, symmetrical disease, early wrist or hip involvement, presence of rheumatoid factor, persistent active disease, and early radiographic changes.

How well did you know this?
1
Not at all
2
3
4
5
Perfectly
46
Q

Heterotopic ossification most commonly develops after arthroplasty of the:

A

The most common postsurgical site for heterotopic ossification after arthroplasty is the hip. The hip is also the most common site of heterotopic ossification occurrence in patients with spinal cord or traumatic brain injury.

How well did you know this?
1
Not at all
2
3
4
5
Perfectly
47
Q

Which stage of complex regional pain syndrome (CRPS 1) is characterized by burning pain, dependent edema, and vasomotor instability?

A

tage 1, the acute phase, is characterized by burning pain, dependent edema, redness, hyperhidrosis, coolness to touch. Stage 2, subacute phase, is characterized by severe pain, fixed edema, and cyanosis or pallor. Stage 3, chronic phase, pain may have subsided, edema subsided, and the extremity is generally stiff. There is no defined 4th stage.

The three clinical stages of type 1 complex regional pain syndrome (CRPS 1) are acute, subacute, and chronic. The acute form lasts approximately 3 months. Pain, often burning in nature, is one of the first symptoms that initially limits function. Swelling, redness with vasomotor instability that worsens with dependency, hyperhidrosis, and coolness to the touch are common physical find-ings. Demineralization of the underlying bony skeleton begins because of disuse.

If the process is not arrested or reversed in the acute phase, the condition may progress to the subacute stage, which can last for up to 9 months. The patient develops persistent severe pain in the extremity and fixed edema that would have been reversible with elevation during the acute phase. The redness of the acute stage gives way to cyanosis or pallor and hyperhidrosis to dry skin. Loss of function progresses, both because of increased pain and fibrosis of the joints caused by chronic inflammation. In the hand, this leads to flexion deformity of the fingers. The skin and subcutaneous tissues begin to atrophy. Demineralization of the underlying bony skeleton be-comes pronounced.

If the process continues, the chronic phase may develop approximately 1 year after disease onset. This stage may last for many years or can be permanent. Pain is more variable during this period. It may continue undiminished or abate. Edema tends to subside over time, leaving fibrosis around the involved joints. The skin is dry, pale, cool, and shiny. Flexion and extension creases are absent. Loss of function and stiffness are marked, and osteoporosis is extreme. In the upper extremity, this can manifest as a frozen shoulder and claw hand.

How well did you know this?
1
Not at all
2
3
4
5
Perfectly
48
Q

What should the serum uric acid level (mg/dl) be lowered to in patients with joint pain symptoms related to gout ?

A

The American College of Rheumatology recommends that the serum uric acid level be lowered to at least 6mg/dl in patients experiencing joint pain related to gout.

How well did you know this?
1
Not at all
2
3
4
5
Perfectly
49
Q

The American College of Rheumatology’s 2010 diagnostic criteria for fibromyalgia includes:

A

The American College of Rheumatology’s 2010 criteria for fibromyalgia uses a Widespread Pain Index (WPI) and Symptom Severity Scale (SSS) in lieu of the older criteria’s reliance on the number of myofascial trigger points present to make the diagnosis. A WPI score greater than 7/19 and a SSS score greater than or equal to 5 is sufficient to diagnosis a patient with fibromyalgia. The presence of major depression, although often comorbid with fibromyalgia, is not necessary for the diagnosis. The exclusion of an underlying rheumatological disease such as rheumatoid arthritis or lupus is not necessary to make a diagnosis of fibromyalgia.

How well did you know this?
1
Not at all
2
3
4
5
Perfectly
50
Q

What percent weight reduction in overweight patients has been shown to result in a moderate to large reduction in self-reported disability related to symptomatic knee arthritis ?

A

Meta-regression analyses show that a 10% reduction in body weight is associated with a moderate or larger clinical improvement in self-reported disability.

How well did you know this?
1
Not at all
2
3
4
5
Perfectly
51
Q

What is the initial deformity created in the cervical spine in a whiplash injury?

A

Whiplash injury initially creates an S-shaped deformation phase in the sagittal plane of the cervical spine, followed by a C-shaped in the sagittal plane phase.

How well did you know this?
1
Not at all
2
3
4
5
Perfectly
52
Q

A 28-year-old female runner presents to you with increasing right foot pain. She recently read an article about barefoot running and so has transitioned to that over the last month. She runs six days a week, but more recently she has had to stop running due to pain and has pain throughout the day, worsened by any weight bearing. On exam, there is no swelling or redness. She has tenderness directly over the 3rd metatarsal. No tenderness elsewhere within the foot. Normal neurological exam. Pain with attempts to hop. For your initial management, you decide to:

A

Her history indicates that she has undergone an abrupt change in her training (barefoot running) that would cause a significant change in the types of forces experienced by her skeletal system, and has an accompanying point of focal bony tenderness. Given this history and exam, the patient likely has a 3rd metatarsal stress fracture. This is not considered a “critical” stress fracture site and can be treated initially with a walking boot. If she is able to walk without limping in 3 weeks, she can be progressed out of the boot and to physical therapy. Non-weight bearing in a cast for 6 weeks is generally not needed with “non-critical” stress fractures and would be overly restrictive. Radiographs are relatively insensitive for stress fractures, and would not likely change management in this case as there is little concern for a “critical” stress fracture. Given her likely injury, pain, and functional limitations, returning her to activities at this time would be inappropriate

How well did you know this?
1
Not at all
2
3
4
5
Perfectly
53
Q

What is the most common form of JIA?

A

Oligoarticular JIA is the most common of the JIA subtypes affecting 50% to 60% of all children with JIA. Polyarticular RF (-) comprises 18-30% of children with JIA. Polyarticular RF (+) occurs in 10% of total JIA cases. Systemic JIA accounts for 2% to 17% of JIA cases.

How well did you know this?
1
Not at all
2
3
4
5
Perfectly
54
Q

Patient is involved in a motor vehicle collision. CT scan shows a fracture at the base of the odontoid process. What is the classification or name of this odontoid fracture?

A

Type 1 odontoid fracture involves a fracture in the tip of the odontoid process and is considered stable. Type 2 fractures are the most common and extend through the base of the odontoid process and are considered unstable with approximately 1/3 of these fractures resulting in nonunion. Type 3 fractures involve fracture of the body of the C2 vertebra and are considered stable. A hangman fracture is traumatic spondylolisthesis of the axis and involves bilateral fractures through the pars interarticularis of the C2 vertebra.

How well did you know this?
1
Not at all
2
3
4
5
Perfectly
55
Q

A 46-year-old healthy male recreational athlete presents to your clinic with a 3-day-old right Achilles tendon rupture. He asks about his treatment options. What do you tell him?

A

Re-rupture rates using functional bracing are similar to re-rupture rates after surgery.

How well did you know this?
1
Not at all
2
3
4
5
Perfectly
56
Q

Known risk factors for congenital hip dysplasia?

A

Breech positioning, female gender, and first born are known risk factors for CHD

How well did you know this?
1
Not at all
2
3
4
5
Perfectly
57
Q

What is the minimum recommended rating of pressure garments to reduce hypertrophic scarring in a patient with a burn injury?

A

To prevent hypertrophic scarring in patients following a burn injury it is advised that compression garments providing at least 25 mmHg of pressure be worn 23 hours per day. Compression garments should be kept clean and replaced routinely to prevent infection and maintain the minimum pressure requirement, respectively. Additional advised skincare includes lubricating the skin several times daily, protection from direct sunlight due to skin sensitivity, and avoidance of excessive heat exposure due to poor thermoregulation related to loss of sweat glands.

How well did you know this?
1
Not at all
2
3
4
5
Perfectly
58
Q

In Clay shoveler’s fracture, what anatomic region of lower cervical and upper thoracic spine is affected?

A

Clay shoveler’s fracture is a repetitive stress injury that affects the spinous process of the lower cervical and upper thoracic spine. In sports, deceleration forces caused by the pull of the trapezius, rhomboids, and the ligamentum nuchae on the neck probably exert repetitive traction on their attachment sites to the narrow spinous processes. The condition is known in manual laborers, but is rare in athletes. It is treated conservatively.

How well did you know this?
1
Not at all
2
3
4
5
Perfectly
59
Q

In addition to mechanical nerve root compression, which of the following is an alternative explanation for symptomatic spinal stenosis?

A

n the theory of venous engorgement, it is believed that the spinal veins of patients with lumbar spinal stenosis dilate due to vascular compression, resulting in venous congestion, interruption in venous flow, and elevated spinal canal pressures. This results in ischemic neuritis of the spinal nerve roots. Signs and symptoms are thought to result from vascular compromise to the vessels supplying the cauda equina (central stenosis) or from pressure on the nerve root complex (lateral stenosis) by the degenerative changes. The clinical impact of these changes is related to the speed by which the compression develops. There have been several hypothesized effects of the focal nerve root constriction: 1) A direct obstruction of the blood flow to the cauda equina; 2) An intraosseous and cerebrospinal pressure change affected by posture; and 3) a direct neuronal compression of the nerve roots.

How well did you know this?
1
Not at all
2
3
4
5
Perfectly
60
Q

How should the hand of a child with juvenile rheumatoid arthritis be positioned in splinting?

A

The correct position for splinting an involved hand in a child with juvenile rheumatoid arthritis is with the wrist in 15° of extension, the metacarpophalangeal joints in 25° of flexion, and the thumb in opposition. This position provides support for weakened structures and helps to reduce contractures.

How well did you know this?
1
Not at all
2
3
4
5
Perfectly
61
Q

Which of the following would typically be specified in a shoe prescription for a patient with a Charcot foot?

A

Rocker-Bottom Sole:

The purpose of custom shoe prescription in someone with a Charcot foot is to minimize the forces the foot sees during standing/ambulatory activity.

When entering the shoe, a large, wide throat assists in reducing trauma to the forefoot and heel by shearing forces.

This can be done by prescribing that the shoe has a Blucher opening, where the bottom of the lace stays are not sewn together across the base of the throat (as is the case with a Balmoral opening).

Using a Velcro strap in place of lacing across the throat reduced the static pressure the upper of the shoe places on the dorsal surface of the foot.

A high and wide toe box reduces pressure around the toes and metatarsal heads (particularly #1 & #5), while a firm heel counter captures the heel to reduce movement of the foot within the shoe, and aids in controlling the shoe during activity.

A custom insert to further relieve static pressure along the plantar surface of the foot and support the longitudinal and mediolateral arches can reduce further orthopedic injury.

Finally, incorporating a rocker bottom into the rubber sole of the shoe will significantly reduce the dynamic pressure experienced by the foot during stance phase by minimizing the plantar area in contact with the ground at any one time.

How well did you know this?
1
Not at all
2
3
4
5
Perfectly
62
Q

recommended static alignment for an uncomplicated transtibial amputee with no significant knee contracture?

A

The socket is prepositioned in 5° of adduction and 5° to 10° of anterior tilt.

The anterior tilt allows loading of the soft tissues of the anterior surface of the residual limb.

The foot is slightly inset relative to the socket.

The patella tendon bar is located halfway between the tibial tubercle and the distal end of the patella.

The posterior brim of the socket should end about an inch below the patella tendon bar, with relief made for the hamstring tendons.

How well did you know this?
1
Not at all
2
3
4
5
Perfectly
63
Q

What is the mechanical advantage of increasing the rear wheel camber of manual wheelchair?

A

Increasing rear wheel camber changes the structure of the wheelchair, which increases the lateral base for increased lateral stability.

The superior aspect of the wheels is narrower, which allows the user increased access to the pushrim and reduces need for increased shoulder abduction.

Changing the rear wheel camber does not have an impact on the anterolateral stability that would impact the need for antitippers.

How well did you know this?
1
Not at all
2
3
4
5
Perfectly
64
Q

What is the primary advantage of a body powered upper limb prosthesis compared to a myoelectric prosthesis?

What are the disadvantages?

A

Primary advantage = Greater sensory feedback

The advantages of body powered upper limb prostheses include the following factors: moderate cost, most durability, highest sensory feedback, and a variety of prehensors available for various activities.

Their disadvantages are that they require the most body movement to operate, have the most harnessing and require increased energy expenditure to use.

Myoelectric prosthesis advantages:
Myoelectric and/or switch controlled upper limb prostheses have the following advantages: They require moderate to no harnessing, require fewer body movements to operate, have moderate cosmesis, provide more function in proximal areas and, in some cases, provide a stronger grasp.

Myoelectric prosthesis disadvantages:
Battery powered prostheses are the heaviest and most expensive prostheses. They also require the most maintenance, provide limited sensory feedback and require extended therapy time.

How well did you know this?
1
Not at all
2
3
4
5
Perfectly
65
Q

What is the advantage of a polycentric knee during the pre-swing (terminal stance) phase of the gait cycle?

A

Many polycentric knees are designed so that the center of rotation moves anteriorly very rapidly during the first few degrees of knee flexion, quickly passing in front of the floor reaction line and facilitating the swing phase.

Because the polycentric knee can be flexed under weight-bearing during the terminal stance, when properly dynamically aligned it can offer both excellent stance stability and ease of swing-phase flexion.

All polycentric knees shorten mechanically to a slight degree during flexion, adding additional toe clearance during midswing.

How well did you know this?
1
Not at all
2
3
4
5
Perfectly
66
Q

What bone is more prone to terminal overgrowth s/p amputation in children?

A

Fibula > Radius

How well did you know this?
1
Not at all
2
3
4
5
Perfectly
67
Q

The primary advantage of an ischial containment socket over a quadrilateral socket in a transfemoral amputee is:

What are the advantages of a quadrilateral socket?

A

Promotion of femoral adduction by distributing the pressure through the socket along the shaft of the femur

An ischial containment socket is designed to stabilize the socket on the residual limb and to control socket rotation by containing the ischial tuberosity and the pubic ramus within the contours of the socket with a snug medio-lateral dimension. This socket has a sub-trochanteric contour that holds the femur in adduction and distributes the pressure through the socket along the shaft of the femur.

Quadrilateral socket:
Flat posterior shelf, ischial set, which provides a primary weight bearing surface for the ischium and gluteal muscles.

Allowance of the glut medius to contract and force the femur into the distal-lateral wall of the socket

How well did you know this?
1
Not at all
2
3
4
5
Perfectly
68
Q

The most effective nonsurgical treatment for de Quervain’s tenosynovitis is:

A

Local corticosteroid injection is proven effective as a treatment for de Quervain’s tenosynovitis, both with and without splinting. Injection alone produced an 83% cure rate, with injection plus splinting producing a 61% cure rate. Splinting alone produced a 14% cure rate, and rest and anti-inflammatories were of no benefit.

How well did you know this?
1
Not at all
2
3
4
5
Perfectly
69
Q

The patellar tendon bearing (PTB) socket for a transtibial amputee is designed with what position/alignment of the knee?

A

The socket is aligned in approximately 5° - 8° of flexion to increase initial tension on the quadriceps tendon (discouraging knee hyperextension) and enhance weight bearing to the anterior aspect of the residual limb.

The PTB socket is designed to accept weight at the patellar tendon, medial flare of the anterior tibia, lateral aspect of residual limb, pretibial muscle mass between the tibial crest and fibula and popliteal fossa.

The posterior wall should provide relief for the hamstring tendons.

How well did you know this?
1
Not at all
2
3
4
5
Perfectly
70
Q

What is the MOST important biomechanical factor for decreasing the vertical loading of the lumbar spine using a TLSO or LSO?

A

Reinforce core musculature to distract vertebral bodies

Abdominal compression increases intracavitary pressure, which acts to unload the spine and its disks by transmitting load onto the soft tissues of the trunk. The application of a 3-point pressure system typically aims to restrict triplanar motion (flexion/extension/lateral flexion and trunk rotation). Pressure over the bony prominences provides a kinesthetic reminder to maintain or correct posture. No spine orthosis achieves complete immobilization of the spine.

How well did you know this?
1
Not at all
2
3
4
5
Perfectly
71
Q

When should a child with congenital radial deficiency be fitted for transradial prosthesis? What is the best initial prosthesis?

A

Children with unilateral transverse radial limb deficiency should be “fit to sit,” meaning fitted by 6 months of age with an initial prosthesis that has a passive terminal device.

How well did you know this?
1
Not at all
2
3
4
5
Perfectly
72
Q

How are mobility devices paid for through Medicare?

A

Medicare part B pays 80% of the allowed purchase price in one lump sum

How well did you know this?
1
Not at all
2
3
4
5
Perfectly
73
Q

What is the positioning of the wrist, MCPs, PIPs, and DIPs when applying a resting wrist-hand-orthosis?

A

This position includes:

-The wrist slightly/moderately “cocked-up” between 10-30 degrees of hyperextension

-The metacarpophalangeal joints mildly flexed

-The proximal and distal interphalangeal joints in extension

-The thumb halfway between palmar and radial abduction.

This position – commonly referred to as a Functional “C” position – reduces the stress on the respective joints, reduces hypertonicity, and permits greater opportunity for functional restoration.

How well did you know this?
1
Not at all
2
3
4
5
Perfectly
74
Q

Why are endoskeletal pylons more advantageous than exoskeletal pylons?

A

Exoskeletal prostheses are more rugged, require less maintenance, cannot be adjusted for alignment after fabrication, and can accommodate only a restricted number of foot and knee units. Furthermore, these prostheses tend to weigh more than the equivalent endoskeletal prostheses. For these reasons, exoskeletal prostheses are prescribed less often than endoskeletal prostheses. Endoskeletal prostheses are modular in design, allowing relative ease of adjustment of alignment and replacement of parts. They are also easier to suspend by virtue of their relatively lighter weight.

How well did you know this?
1
Not at all
2
3
4
5
Perfectly
75
Q

In patients with knee arthritis affecting mostly the medial compartment, which shoe insert will most effectively reduce the knee adduction moment?

A

A lateral heel wedge can reduce the adduction moment at the knee and potentially reduce loading of the medial knee compartment. This may help reduce pain and improve function in some patients with knee arthritis.

How well did you know this?
1
Not at all
2
3
4
5
Perfectly
76
Q

The primary stabilizing effect of the flexible lumbosacral orthosis is its ability to:

A

Elevate intra-abdominal pressure

Although they do not effectively restrict motion to a significant degree, flexible lumbosacral orthoses elevate intra-abdominal pressure, thereby unloading the spine and supporting structures. This action also provides inhibitory kinesthetic feedback and warmth. Long-term use of binders and other flexible lumbosacral orthoses may, unfortunately, result in atrophy of trunk muscles.

How well did you know this?
1
Not at all
2
3
4
5
Perfectly
77
Q

What is the proper fighting height of a standard or rolling walker?

A

When properly fit, the height of the grips of a walker or cane should be near the level of the greater trochanter of the hip; this permits a 20 degrees of flexion in the elbow.

This also corresponds to the position of the ulnar styloid with the arms in a comfortable position.

A rolling or standard walker, in use for normal ambulation, is not meant to permit full weight bearing, but rather improve balance by providing a larger base of support.

The proper height of an assistive ambulatory device does not change based upon its intended use.

How well did you know this?
1
Not at all
2
3
4
5
Perfectly
78
Q

Indicators for poor prognosis for upper limb motor recovery after a stroke include:

A

Complete arm paralysis at onset

No measurable grasp strength at 4 weeks

Severe proximal spasticity

Severity of weakness at onset of stroke has been demonstrated to be the most important predictive factor.

How well did you know this?
1
Not at all
2
3
4
5
Perfectly
79
Q

The Bobath and Brunnstrom Approach to movement therapy differ primarily in their approach to:

A

Primitive movements

How well did you know this?
1
Not at all
2
3
4
5
Perfectly
80
Q

Constraint induced therapy (CIMT) requires that patients:

A

Participants in constraint induced therapy (CIMT) are required to be able to voluntarily extend their wrists and move their fingers in order to participate in this therapy. Therapy is usually initiated within 2 weeks of onset but not usually within 24 hours. Most CIMT protocols require constraint of the unaffected limb 90% of waking hours.

How well did you know this?
1
Not at all
2
3
4
5
Perfectly
81
Q

Following a stroke, deep tendon reflexes increase within:

A

Immediately following onset of hemiplegia, there is a loss or decrease in tendon reflexes. Increased deep tendon reflexes on the involved side develops within 48 hours.

How well did you know this?
1
Not at all
2
3
4
5
Perfectly
82
Q

Infarction of which blood vessel causes ipsilateral facial and contralateral body loss to pain and temperature sensation, Horner’s syndrome, ataxia, vertigo and dysphagia?

A

The symptoms given are classic for Wallenberg Syndrome, also known as Lateral Medullary Syndrome. This may be caused by occlusion of the Posterior inferior cerebellar artery (PICA), vertebral arteries, and lateral medullary arteries (superior, middle and inferior).

How well did you know this?
1
Not at all
2
3
4
5
Perfectly
83
Q

What imaging modality is used to measure size of a hemorrhagic stroke and and has also been shown to be a strong predictor of outcome?

A

CTH

How well did you know this?
1
Not at all
2
3
4
5
Perfectly
84
Q

In the setting of a high clinical suspicion for subarachnoid hemorrhage, with a negative CT scan, what diagnostic test should you perform next?

A

A spinal tap for evaluation of cerebral spinal fluid is the recommended exam to follow a negative non contrast CAT scan with high clinical suspicion of subarachnoid hemorrhage.

How well did you know this?
1
Not at all
2
3
4
5
Perfectly
85
Q

When is CT angiogram indicated in the setting of a stroke?

A

CT Angiogram is indicated to evaluate extracranial carotid stenosis, and internal cerebral circulation for identifiable vascular defects.

How well did you know this?
1
Not at all
2
3
4
5
Perfectly
86
Q

Which factor is associated with improved outcomes in hemorrhagic stroke patients?

A

The volume of hemorrhage is a strong predictor of outcome in hemorrhagic stroke patients. Patients with more than 5cm of hemorrhage in the basal ganglia had certain mortality by 30 days poststroke.

Studies demonstrate that persons who score higher than 9 on the Glasgow Coma Scale, and who require intracranial pressure (ICP) monitoring and experience persistent elevation of ICP are more likely to have herniation and poorer outcomes.

Aggressive antihypertensive management in the setting of intracerebral hemorrhage (ICH) remains controversial; consensus guidelines recommend systolic blood pressure above 200mmHg be treated with intravenous antihypertensives initially in order to not further extend the hemorrhage.

Cerebral perfusion pressure (CPP) should be maintained between 60-80mmHg when systolic blood pressure is above 180mmHg and intracranial pressure (ICP) is within normal range (8-18mmHg).

How well did you know this?
1
Not at all
2
3
4
5
Perfectly
87
Q

Your stroke patient has weakness affecting the hand and foot more than the proximal muscles on the same side. Sensation and cognition are intact. This person most likely has a stroke in which contralateral area?

A

Internal Capsule

Stroke in the brainstem would result in cranial nerve involvement.

With a midbrain stroke there would be additional findings such as decreased coordination, cranial nerve involvement, and sensory deficits.

With motor cortex involvement the patient would have extensive weakness in the arm or leg depending on the location of the stroke. Also with a cortical stroke there would be some cognitive and speech involvement.

How well did you know this?
1
Not at all
2
3
4
5
Perfectly
88
Q

Hemorrhagic strokes caused by hypertension are typically located in the:

A

Hemorrhagic strokes account for about 15 % of all strokes and are most often caused by hypertension. The lesions are typically located in the basal ganglia, thalamus, pons, and cerebellum.

How well did you know this?
1
Not at all
2
3
4
5
Perfectly
89
Q

A patient has right 3rd cranial nerve palsy, left side loss of pain and temperature, left side loss of joint position, and left side ataxia. Where is the brainstem stroke most likely located?

A

The characteristics of the findings are most consistent with a brainstem stroke in the region of the tegmentum of the midbrain (Benedikt syndrome).
This affects the 3rd cranial nerve, spinothalamic tract, medial lemniscus, superior cerebellar peduncle, and the red nucleus (this can lead to contralateral chorea).

The medial basal midbrain (Weber syndrome) affect the 3rd cranial nerve and the contralateral corticospinal tract resulting in hemiplegia.

The lateral pons (Millard-Gubler syndrome) involves the 6th and 7th cranial nerves and the corticospinal tract.

The lateral medulla (Wallenburg syndrome) involves the 5th cranial nerve, spinocerebellar tract causing ipsilateral ataxia, spinothalamic tract, vestibular nuclei causing nystagmus, sympathetic tract resulting a Horner syndrome, and nucleus ambiguus causing dysphagia and dysphonia.

How well did you know this?
1
Not at all
2
3
4
5
Perfectly
90
Q

An infarct anterior to the precentral gyrus within the frontal lobe is most likely to result in which one of the following deficits?

A

Anterior to the precentral gyrus in the frontal lobe is the premotor area, which is important in motor planning. Injury to this area can cause poor static and dynamic balance as well as movement disorders

How well did you know this?
1
Not at all
2
3
4
5
Perfectly
91
Q

Wernicke type of apahasia is due to injury in the

A

Wernicke type of apahasia is due to injury in the posterior superior portion of the first temporal gyrus near the primary auditory cortex.

How well did you know this?
1
Not at all
2
3
4
5
Perfectly
92
Q

An 80-year-old man presents with mild expressive aphasia and slight right sided weakness. Symptoms resolve within 15 minutes. He is diagnosed with a transient ischemic attack. Carotid ultrasound demonstrates 70%-99% occlusion of the left internal carotid artery (ICA) and 50%-70% occlusion of the right internal carotid artery. What is the most appropriate recommendation regarding carotid endarterectomy?

A

Consideration of carotid endarterectomy (CEA) involves several factors, including whether the patient is symptomatic or asymptomatic and the degree of internal carotid artery (ICA) stenosis.

For symptomatic patients with transient ischemic attack or mild stroke, CEA is recommended if ICA stenosis is 70% to 99% but should only be performed in select patients if ICA stenosis is 50% to70%.

The CEA procedure is not recommended for symptomatic patients if ICA stenosis is less than 50%. For asymptomatic patients, CEA may be considered if ICA stenosis is 60% to 99%, but typically is recommended only if ICA stenosis is greater than 70% to 80%, if the patient is medically stable and is expected to live 5 years or longer, and if performed in a center with less than 3% perioperative complication rate.

How well did you know this?
1
Not at all
2
3
4
5
Perfectly
93
Q

Botulinum toxin A works by cleaving which presynaptic translocation protein?

A

SNAP 25; Botulinum toxin A works by cleaving the SNAP 25 translocation protein. Other botulinum toxin subtypes cleave the VAMP/SYB2, syntaxin, and gamma SNAP proteins.

94
Q

What is the most common cause of mortality after total hip replacement?

A

The most common cause of mortality after total hip replacement is cardiovascular complications followed by thromboembolic events.

95
Q

Immunodeficiency states, both congenital and acquired, increase one’s risk for developing which of the following conditions?

A

Primary central nervous system lymphomas (PCL) is a variant of extranodal non-Hodgkin lymphoma that involves the brain, leptomeninges, eyes, or spinal cord without evidence of systemic disease. It accounts for 2% to 3% of cerebral tumors. Immunodeficiency increases the risk of developing this type of tumor although it can be seen in immunocompetent individuals.

96
Q

Leptomeningeal disease is:

A

Tumors of the spinal axis occur in three spaces. Intramedullary tumors are in the parenchyma of the spinal cord and as such are also contained by the dura and thus intradural. Epidural tumors are outside the dura and can also be paraspinal. Leptomeningeal disease occurs within the dura but outside the spinal cord and is therefore also extramedullary.

97
Q

Pain associated with bone metastasis is associated with:

A

Bone marrow is insensitive to pain and metastasis confined to the marrow is rarely painful. Painful metastatic lesions usually involve the periosteum, paravertebral soft tissues, or nerve roots before they become symptomatic.

98
Q

What is a common physiologic change that pts experience s/p lung transplant?

A

Denervation of the transplanted lung results in an impaired cough reflex and ineffective secretion management. Post-operative rehabilitation must therefore focus on strategies to improve cough and secretion mobilization. Recipients of lung transplantation are typically able to sufficiently participate in an exercise program and achieve exercise benefits such as improved endurance and strength at rates comparable to their age-matched healthy controls. However, maximal oxygen consumption usually remains reduced (32-60% of predicted). Immobility should be avoided post-transplantation. Post-transplant rehabilitation should start in the intensive care unit on day 1 with range of motion exercises and progress to transfers and ambulation as soon as possible. Rehabilitation is similar to that used for chronic obstructive pulmonary disease, with focus on strengthening, conditioning, education, pulmonary toilet, and medication and oxygen use.

99
Q

The most common sign of metastatic brain cancer seen on physical examination is:

A

Hemiparesis is appreciated in 59%, impaired cognitive function in 58%, hemisensory loss in 21%, papilledema in 20% and visual field cut in 7%.

100
Q

In women with breast cancer and upper extremity lymphedema, resistance training results in:

A

Decreased lymphedema symptom severity

Progressive resistive training in breast cancer survivors with lymphedema has been demonstrated to NOT worsen lymphedema volumes relative to controls (breast cancer patients who did not participate in progressive resistive training).

However, progressive resistive training in breast cancer survivors has NOT been demonstrated to improve lymphedema volumes.

Progressive resistance training has been demonstrated to decrease lymphedema exacerbations as well as the amount and severity of lymphedema symptoms compared to controls. Both upper and lower-body strength is also increased.

101
Q

Which physiologic changes in cardiac function occur after heart transplantation?

A

Following orthotopic cardiac transplantation, stroke volume may be reduced due to diastolic dysfunction from increased myocardial stiffness in the new heart. Because of vagal denervation, resting tachycardia near 100 beats per minute is frequently observed. Similarly, peak heart rate is 20% to 25% lower than age matched controls, as the heart rate and blood pressure response to exercise is blunted, and the allograft requires the effects of circulating catecholamines to increase stroke volume and HR in response to exercise.

102
Q

How can you ilicit a murmur on physical exam in a patient with HOCM?

A

Hypertrophic cardiomyopathy (HCM) is due to asymmetric left ventricular wall thickening. This creates a dynamic outflow obstruction typically due to systolic anterior motion of the Mitral valve. There is a genetic component related to a sarcomere gene mutation. A murmur due to HCM will increase in intensity with any maneuver that decreases the volume of blood in the left ventricle (such as standing abruptly or a Valsalva maneuver) as opposed to a flow murmur which would decrease with Valsalva.

103
Q

The American Geriatrics Society’s goal of diabetic control in an otherwise healthy elderly individual includes:

A

American Geriatric Society guidelines place greater emphasis on reduction of cardiovascular complications and minimizing the risk of hypoglycemia. In an otherwise healthy elderly individual, fasting plasma glucose below 130 mg/dL and a hemoglobin A1c of less than 7.5% is sought.

104
Q

The most common location for metastatic brain tumors is:

A

Cerebral hemisphere

105
Q

What would be expected findings in a patient following cardiac transplant:

A

Resting tachycardia with delayed chronotropic response to exercise

106
Q

Peak VO2 decreases with age, declining per decade by:

A

Peak VO2 declines by 8% to 10% per decade.

107
Q

What is the most common cause of spinal cord dysfunction in patients with HIV?

A

Vacuolar myelopathy

108
Q

Brain metastases in both men and women are most commonly caused by which primary cancer?

A

Almost 50% of brain metastases are derived from lung cancer. Breast cancer is the second most common source, followed by melanoma.

109
Q

Relative contraindications to pulmonary rehab program

A

However, the patient cannot have an acute medical, orthopedic or neurologic condition that impedes exercise. Lastly, patients should be motivated to complete the exercise programs and abstain from tobacco products.

110
Q

risk factor that is associated with development of a venous thromboembolism (VTE) in a child with traumatic brain injury (TBI).

A

TBI due to non-accidental trauma

111
Q

In adults with myelomeningocele, what percentage of individuals report consistent bowel continence?

A

Approximately 55% of adults (older than 20 years) with myelomeningocele report consistent bowel continence. Bowel continence has been positively associated with quality of life, employment, and educational attainment.

112
Q

In the treatment of lymphedema, which condition is a relative contraindication to performing complete decongestive therapy?

A

Relative contraindications to complete decongestive therapy include significant congestive heart failure, acute deep venous thrombosis, acute or untreated infection or inflammation of the affected limb, and fracture. Lymphatic drainage massage should be avoided over concurrently irradiated soft tissues, but a history of previous radiation therapy is not a contraindication for therapy.

113
Q

Which of the following is correlated with postoperative cognitive deficits after coronary artery bypass graft surgery (CABG)?

A

Each additional hour on cardiac bypass doubles the probability of postoperative encephalopathy. Lesions demonstrated on diffusion weight magnetic resonance imaging correlate poorly with impairment. Impairments may be transient, with recovery at 8 weeks predictive of cognitive function at 5 years. Cognitive deficits are most prominent at 3 days post CABG.

114
Q

A 10-year-old boy presents with fever, knee pain, and hematuria. Physical exam reveals a malar rash, pericardial rub and bilateral knee effusions. His bloodwork shows leukopenia and Anti-c1q autoantibodies. Involvement of which organ involvement determines the long-term survival?

A

Kidney function:

Childhood Systemic Lupus Erythematosus (cSLE) has a more severe disease course and higher mortality than adult SLE with increased rates of aggressive renal disease, hematologic anomalies, photosensitivity, and neuropsychiatric and mucocutaneous involvement. Skin involvement in cSLE is common at onset and during the disease course. The malar, or erythematous “butterfly” rash, is the most typical cutaneous manifestation; it may occur after exposure to sunlight. Nonscarring frontal alopecia is often seen in cSLE. At disease onset, there is often a combination of fever, weight loss, arthralgia, photosensitive or malar rash, and renal disease. Lupus nephritis is an important predictor of long-term survival, especially if not controlled early. Anti-c1q autoantibodies are high risk for the development of renal disease; vigilant monitoring of renal function every three months is recommended, especially in males with cSLE.

115
Q

In individuals with intermittent claudication, exercise improves which of the following?

A

Walking distance:

There is high quality evidence that an exercise program increases pain-free and maximum walking distance in individuals with intermittent claudication. There is no evidence that exercise improves ABI, mortality or risk of amputation.

116
Q

The decrease in cardiac mortality from participation in a program of cardiac rehabilitation is:

A

25%;

Comprehensive cardiac rehabilitation programs that address reducing risk factors and lifestyle changes such as nutrition, weight loss and smoking cessation have an impact on cardiac mortality by reducing risk by 25%.

117
Q

Which phase of cardiac rehabilitation is a class 1 recommendation from the American Heart Association and American College of Cardiology after myocardial infarction?

A

Referral to outpatient phase II cardiac rehabilitation is a class 1 recommendation from the American Heart Association and American College of Cardiology after myocardial infarction, coronary vascularization, valvular heart surgery, heart transplant, and in stable heart failure with reduced ejection fraction

118
Q

Which patient population meets the criteria for approved payment for cardiac rehabilitation by the Centers for Medicare and Medicaid Services?

A

HF with EF 30%, 7 weeks post hospital discharge

119
Q

Bookmark
User Note
Which electrodiagnostic finding is most compatible with early stage anterior horn cell disease?
A. Small sensory responses
B. Normal CMAP studies
C. Small amplitude motor unit potentials
D. Prolonged somatosensory evoked potentials

A

A. Normal motor conduction studies

In anterior horn cell diseases such as amyotrophic lateral sclerosis (ALS), the sensory nerves are not usually affected. In the early stages of the condition motor nerve conductions can be normal. Because ALS is a neuropathic condition with ongoing denervation and reinnervation, one would not expect to find small amplitude motor units with early or increased recruitment; that pattern represents a myopathic process.

120
Q

75-year-old female presents to clinic with a large, cool synovial effusion of her right shoulder. Physical examination is significant for pain-limited range of motion (ROM) and profound rotator cuff muscle weakness. X-rays of the patient’s shoulder reveal superior subluxation of the humeral head, extensive bony destruction of the glenohumeral joint, soft-tissue effusion, and calcific deposits. The most likely diagnosis is:

A

Milwaukee Shoulder AKA pseudogout of the shoulder.

MSS is a destructive, calcium phosphate crystalline arthropathy; it encompasses an effusion that is noninflammatory with numerous aggregates of calcium hydroxyapatite crystals in the synovial fluid, associated with rotator cuff defects . Calcium hydroxyapatite crystal disease is characterized by recurrent painful periarticular calcific deposits in tendons, soft tissues, or intra-articular surfaces. The pathophysiology is believed to be the intra-articular calcium hydroxyapatite deposition. MSS occurs in elderly patients typically aged 60–90 years. There is a female preponderance in the ratio of 4 : 1. Radiographic changes on plain X-ray show joint space narrowing, subchondral sclerosis with cyst formation, destruction of subchondral bone, soft-tissue swelling, capsular calcifications, and intra-articular loose bodies.

121
Q

Which of the following may result from pistoning in a transtibial socket?

A

Pistoning of the residual limb within a prosthetic socket refers to the sliding of the limb into the socket during stance phase, and out of the socket during swing phase (specifically, terminal stance). This occurs due to a lack of suspension, such as wearing too few donning socks (insufficient ply) or having a hole in the suspension sleeve, thus breaking suction.

As a result of the limb sliding into the socket, there is commonly pain at the inferior patella and antero-distal end of the limb from repetitively striking of the patellar tendon bar and distal end of the socket, respectively. When the amputee’s weight is placed onto the prosthesis during early stance phase, the hip on that side will lower (e.g., dip) as they sink into the socket.

As weight is transferred off the prosthesis during terminal stance, the limb will partly slide out of the socket, resulting in a relatively longer limb. To clear this limb during swing phase, the amputee will hike their hip on the prosthetic side and will often circumduct the prosthesis to avoid tripping over the foot.

122
Q

A patient who had a right MCA stroke presented one hour after symptom onset and received intravenous thrombolysis with t-PA, improving his NIHSS from a 12 to a 5. He is ambulating with physical therapy on the acute medical floor. Which factor has greatest evidence to predict his chances of going home?
A. Premorbid functional status
B. tPA within 3 hrs of sxs
C. Early mobilization within two days
D. Anatomic location of stroke

A

Patients who receive intravenous thrombolysis for acute ischemic stroke are more likely to be discharged directly home after hospitalization with outpatient neurorehabilitation therapies. There is no correlation with the exact time within the three-hour window. His premorbid functional status, location of stroke, and early mobilization are important components of his rehabilitation; however, they have no bearing on the efficacy of t-PA therapy.

123
Q

In the severe TBI population, the possibility of good recovery is unlikely when post-traumatic amnesia (PTA) exceeds:

A

3 months

Lower GCS scores are associated with worse outcome. Severe disability is unlikely when length of coma is < 2 weeks and good recovery is unlikely when coma > 4 weeks.

Severe disability is unlikely when PTA is < 2 months and good recovery is unlikely when PTA > 3 months.

Good recovery is unlikely when patient is >65 y/o.

Good recovery is unlikely when bilateral brainstem lesions are present on early MRI.

124
Q

A 47-year-old injured worker with low back pain is near maximum medical improvement (MMI) and is referred for a functional capacity evaluation (FCE). What knowledge can be obtained from FCEs for injured workers with chronic low back pain?

A

Performance on the floor-to-waist lift are as predictive as the number of failed tasks in the entire FCE protocol. Limitations noted in an FCE apply to occupational and non-occupational (eg, recreational) activities. FCEs are used to estimate a patient’s functional abilities, whereas work-hardening programs are designed to increase a patient’s functional abilities. Following an FCE, 20% of patients experience back-related events within 1 year.

125
Q

What is the most common cause of emergency room visits for traumatic brain head injuries?

A

Falls;

In 2014, falls became the leading cause for TBI. Falls account for 48% of all TBI related emergency department room visits. Falls disproportionately affect children and older adults. Being struck by or against an object is the second leading cause of TBI-related emergency department visits. Falls and motor vehicle crashes were the leading causes of all TBI-related hospitalizations.

126
Q

A 63-year-old patient with amyotrophic lateral sclerosis presents to your clinic with insidious onset of dyspnea over the last several months. What is the most likely abnormality on pulmonary function testing?

A

Reduced Tidal volume

In restrictive lung disease, the primary abnormality is low tidal volume. In extrinsic restrictive lung disease (neuromuscular disease including amyotrophic lateral sclerosis, paralysis and kyphoscoliosis), this is due to weakness of the chest wall muscles. In intrinsic restrictive lung disease (pulmonary fibrosis, sarcoidosis), this is due to noncompliant lung tissue. In obstructive lung disease (asthma, COPD), abnormalities include increased residual volume and reduced forced expiratory volume due to incomplete exhalation or obstruction.

127
Q

What is the most sensitive neuroimaging technique to detect diffuse axonal injury after traumatic brain injury?

A

Diffusion-weighted MRI scan

Standard computed tomography and magnetic resonance imaging (MRI) scans are often unremarkable after isolated diffuse axonal injury (DAI) in patients with significant cognitive and behavioral problems. However, fluid-attenuated inversion recovery (FLAIR), diffusion-weighted and gradient echo MRI sequences are more sensitive in detecting evidence of DAI after a TBI. The T2 weighted image is more sensitive to intraparenchymal blood than is FLAIR and can reveal hemorrhagic DAI, it also provides greater detail on brainstem and central gray matter.

128
Q

Which of the following finger deformities common in patients with rheumatoid arthritis is caused by synovitis of the metacarpophalangeal (MCP) or distal interphalangeal (DIP) joints creating a flexion contracture of the MCP joint, hyperextension of the PIP joint, and flexion of the DIP joint?

A

Swan Neck Deformity

129
Q

Which electroencephalogram pattern is associated with a better prognosis after traumatic brain injury?

A

Favorable electroencephalogram (EEG) patterns after a traumatic brain injury are normal activity, rhythmic theta activity, frontal rhythmic delta activity, and spindle pattern.

Poor prognosis is associated with epileptiform activity, nonreactive, low amplitude delta activity and burst suppression patterns with interruption of isoelectricity. Complete isoelectric EEG activity had the highest mortality.

130
Q

What is the most common etiology of low back pain in a 13-year-old female athlete?

A

Spondylolysis

131
Q

A college basketball player present with knee pain and swelling after he suffered an injury to his knee while coming down from a rebound with an awkward landing. On initial radiographs, you find an avulsion fracture off the lateral tibial plateau. What structure is most likely to be injured?

A

Segond Fracture; which is considered pathognomonic for the presence of an anterior cruciate ligament (ACL) tear.

132
Q

Which of the following is commonly used as an outcome measure to assess perception of physical and emotional functioning in patients with chronic pain?

A

Short Form 36 (SF-36)

The SF-36® and the shorter SF-12® is a measure of quality of life with questions covering multiple domains. SF-36 Domains include “physical functioning; role limitations due to physical health; role limitations due to emotional problems; energy/fatigue; emotional well-being; social functioning; pain; general health”. These questionnaires are owned and marketed by Medical Outcomes Trust. They have a considerable research base and assess a broad range of activities of daily living relevant to disability.

133
Q

A 65-year-old woman complains of chronic back pain for “decades” and increased difficulty with movement. There is no history of recent trauma. Radiographs of her thoracolumbar spine are taken and demonstrate relative preservation of disc height along with continuous calcifications along the anterolateral areas. The sacroiliac joints are unremarkable. What is her diagnosis?

A

Diffuse idiopathic skeletal hyperostosis

Radiographs of diffuse idiopathic skeletal hyperostosis (DISH) demonstrate calcification of the anterior longitudinal ligament. Criteria for DISH include the relative preservation of intervertebral disk height, flowing anterolateral calcifications of at least 4 continuous vertebral levels, and the absence of sacroiliac joint erosion

134
Q

Cardiopulmonary exercise testing is used to evaluate:

A

Aerobic Capacity

While estimates can be made of cardiac stroke volume, muscle strength and sprint velocity, aerobic capacity is what is measured by cardiopulmonary exercise testing. Cardiac ischemia may be detected by the 12-lead monitoring used on cardiopulmonary exercise testing (CPET), but CPET is a functional test, not a diagnostic test like cardiac exercise testing or the variants such as thallium, persantine, or other tests.

135
Q

Clinical symptoms and signs of tethered cord syndrome include:

A

The most common clinical signs or symptoms of a tethered cord include spasticity in the lower extremities, decline in lower extremity strength, and worsening scoliosis. Other signs and symptoms that strongly suggest tethering of the spinal cord include back pain, changes in urologic function, changes in gait, changes in sensation, and development of lower extremity contractures.

136
Q

Which measure often used to identify the existence of comorbid psychological problems present in chronic pain patients and to help tailor treatment?

A

Minnesota multiphasic personality inventory (MMPI)

The MMPI and its variants (version 2 and 2RF) are the most widely used psychometric tests for adult personality and psychopathology testing.

137
Q

Juvenile idiopathic arthritis (JIA) can be differentiated radiographically from ankylosing spondylitis (AS) by the:

A

Constellation of cervical facet joint ankylosis and vertebral body hypoplasia in patients with JIA

By definition, JIA affects the immature, growing skeleton. Diffuse ankylosis of the cervical facet joints in the setting of vertebral body and disc hypoplasia is virtually diagnostic. In contrast, AS affects the mature skeleton after the intervertebral discs have formed. The cervical spine is more commonly involved in patients with JIA than other spinal levels with AA instability being the most common radiographic finding. Enthesopathy and tarsal disease in the first year of presentation is diagnostic for JIA even in the absence of axial disease

138
Q

Which of the following aids in prognosticating outcomes of TBI at 6 months post injury?

A

Magnetic resonance spectroscopy (MRS) provides information about the neurochemical status of the brain and may indicate the extent of brain damage; therefore, it provides important data for evaluating expected outcome. Studies found that MRS studies correlated with outcomes 6 months later. Furthermore, several studies found threshold levels for the metabolites that also correlate with prognosis. Positron emission tomography (PET) uses radioactive tracers that localize to metabolically active regions. PET scanning is more sensitive in detecting abnormalities after traumatic brain injury than is standard neuroimaging, although PET has poor specificity and has not been studied in regard to prognostic value. Earlier studies suggest that the presence of apolipoprotein-E4 allele (APOE-e4) would indicate a worse functional outcome, but that finding has not been replicated. There are conflicting reports on APOE-e4. It is unlikely that a patient’s APOE-e4 status will play a role in prognostication at the present time. Transcranial magnetic stimulation is useful in examining cortical motor representation after cerebral injury and may be useful in treatment of TBI but has not been studied in regard to prognostic value.

139
Q

Which salivary glands should be targeted with botulinum toxin to control anterior drooling when there is no food stimulation?

A

Submandibular

Since the submandibular glands are the major producers of saliva when the patient is not stimulated by food, and parotid glands produce mainly during feeding or other oral motor stimulation, these are the glands to target. Because the sublingual glands produce less than 5% of saliva, they typically are not injected. The buccal glands are located in the mucous membrane lining the cheeks and mouth. These glands produce only a small amount of saliva. The parotid glands can be localized by surface anatomy, whereas the submandibular glands may require ultrasound guidance. Current studies show significantly decreased anterior drooling after botulinum toxin A injections to the submandibular and parotid glands. It should be noted that use of botulinum toxin A (Botox) is off-label for those under 18 years of age.

140
Q

The Galveston Orientation and Amnesia Test (GOAT) and Orientation Log (O-Log) are standard measures for assessing post-traumatic amnesia (PTA). A patient is no longer in PTA when:

A

The GOAT score can range from 0 to 100, with a score of 75 or better defined as normal. The end of PTA can be defined as the date when the patient scores 75 or higher in GOAT for two consecutive days. The O-Log score can range from 0 to 30, with a score of 25 or better on two consecutive days defined as the end of PTA.

141
Q

Which clinical test is most sensitive and specific for an anterior cruciate ligament tear?

A

Lachman

142
Q

Which of the following is good practice to prevent electrical injury during electrodiagnostic studies?

A

Avoid extension cords

143
Q

Which of the following factors predicted the highest risk for disability at one year following low back injury?
A. Radicular Symptoms
B. Severity of Injury
C. Sedentary Job
D. Initial chiropractic treatment

A

A. Radicular Symptoms

Per Official Disability Guidelines, “workers with more severe back injuries were more likely to be on disability after one year. Employees with pain spreading down into the leg indicating radiculopathy were at particularly high risk. Another significant predictor was the specialty of the first doctor seen after the back injury, and workers who saw a chiropractor were less likely to be disabled at one year. Certain job characteristics also affected disability risk, and risk was lower when employers offered modified duty. All of the significant factors were combined into a statistical model which was 88 percent accurate in identifying workers who would and would not be disabled after one year. In this study most of the psychological factors thought to contribute to chronic pain were not significant risk factors for disability.

144
Q

In patients with a history of ventricular arrhythmias and ischemic heart disease, exercise stress tests are:

A

Used to determine a safe target HR that does not provoke arrhythmias

Exercise stress tests are used to screen for ventricular arrhythmias and to determine the target heart rate, which is set below the level at which arrhythmias are noted. Upright exercise produces less myocardial oxygen demand than supine exercise and therefore patients prone to arrhythmias should be advised to exercise in the upright position. Approximately 80% of patients with a history of ventricular arrhythmia will have a ventricular arrhythmia during inpatient cardiac rehabilitation

145
Q

12-year-old ballerina presents to your clinic with right ankle pain. She was dancing pain-free until this year when she began to dance on pointe. On examination, you note tenderness to palpation at the posterior talocrural joint, pain with end passive ankle plantarflexion range of motion, and no pain with resisted active ankle dorsiflexion or plantarflexion. Which of the following would you anticipate with imaging?

A

Os trigonum on ankle radiographs:

Dancers who go on pointe are at risk for posterior ankle impingement syndrome, which results from impingement of the posterior aspect of the talus between the tibia and calcaneus with repetitive ankle plantarflexion. One predisposing factor for this condition is the presence of an os trigonum, which results from an accessory ossification center of the posterior talus. While Achilles tendinopathy and peroneal tendon injuries are not uncommon in ballet dancers, the patient’s location of pain and physical examination are not indicative of either of these as the diagnosis. Similarly, her exam is not suggestive of Haglund’s deformity; furthermore, this patient would be relatively young for this condition.

146
Q

During Pre-swing in a normal individual’s gait cycle, which muscle is most active?

A

Ipsilateral iliopsoas muscle

The purpose of Pre-swing during the gait cycle is to prepare the limb to propel forward during swing phase. During this phase, the iliopsoas muscle of the ipsilateral limb is contracting concentrically to flex the hip for momentum. The contralateral gluteus medius muscle doesn’t start to contract until initial swing, once the foot has left the ground, to stabilize the pelvis (prevent it from dropping). Neither the gluteus maximus or pretibial muscles are active during preswing; the ipsilateral gluteus maximus is eccentrically active only during loading response to prevent the trunk from rotating forward (from momentum), and the pretibial muscles do not start to contract until the foot leaves the ground during initial swing, to prevent foot drop.

147
Q

Which factor is a criterion for hip osteoarthritis?

A

The American College of Rheumatology states that the criteria for osteoarthritis of the hip are hip pain along with two of the three findings: erythrocyte sedimentation rate less than 20mm/hr, radiographic evidence of femoral/acetabular osteophytes, or radiographic evidence joint-space narrowing. Periarticular osteopenia and femoral head erosions are more likely to be seen in patients with rheumatoid arthritis.

148
Q

A 3-year-old male presents with toe walking and elevated transaminases. What additional physical exam finding is most likely?

A

Elevated transaminases are frequently found incidentally in boys with Duchenne muscular dystrophy (DMD). This is due to muscle breakdown as indicated by a significantly elevated CK level. Exam is notable for calf pseudohypertrophy.

149
Q

Which measure evaluates patients’ self-perception of disability specifically for low back pain?

A

Oswestry Disability Index

150
Q

A 5 year-old boy with spina bifida has full hip flexion against gravity and his knee extension strength is at least 4/5. The child has ankle dorsiflexion but not plantarflexion. His feet are in neutral position. Hip examination is symmetric. What will most likely be this child’s primary means of mobility as he grows older?

A

This child has a strong quadriceps muscle and no deformities noted at 6 months of age. He is reported to be healthy. Sitting balance and neurologic level were good predictors of ambulation potential. The best early predictor of ambulation in children with spina bifida is a strong quadriceps muscle. Negative predictors are spine and lower extremity deformities and obesity. Children do not typically learn to use crutches until 3 to 5 years of age or older.

151
Q

A hinged/articulated AFO with plantar flexion stop and free dorsiflexion would best be prescribed in which of the following scenarios?

A

Hinged or articulated AFOs with plantar flexion stop is indicated when there is sufficient hindfoot dorsiflexion and strength that will allow movement of tibia over the foot in stance. They may be used in the presence of milder hypertonia or spasticity. It may promote crouched posture by allowing increased dorsiflexion and knee flexion

152
Q

A 53-year-old woman was treated for Hodgkin lymphoma with 3600 cGy of mantle field radiation when she was 23 years old. She now presents to your clinic with progressive difficulty holding her head erect and upper extremity weakness. Which of the following is LEAST likely to be contributing to her symptoms?

A

Mantle field radiation includes all the lymph nodes in the neck, chest, and axilla. All structures in the radiation field are subject to damage, which can become clinically evident years later and can progress indefinitely. In addition to viscera such as the heart and lungs, mantle field radiation can damage all neuromuscular structures in the field including the spinal cord, nerve roots, plexus, local nerves, and muscles. This has been termed a “myelo-radiculo-plexo-neuro-myopathy.” The term “polyneuropathy” describes a diffuse neuropathic process affecting peripheral nerves. The damage to named and unnamed peripheral nerves from focus radiation is confined to the radiation field (i.e., multiple mononeuropathies) and therefore this term is not appropriate.

153
Q

Radiographic findings associated with ankylosing spondylitis include symmetric SI joint narrowing, osteopenia, and a “Bamboo spine.” A “Bamboo spine” refers to ossification of which structure?

A

Annulus Fibrosis

This patient has ankylosing spondylitis. The hallmark is bilateral sacroiliitis. The formation of a bamboo spine on radiographic studies is secondary to ossification of the annulus fibrosis leading to bridging syndesmophytes. Interspinous ligament ossification can result in a “dagger sign” on xray.

154
Q

A patient with multiple myeloma presents with a new pathologic fracture of the right superior pubic ramus. Computed tomography scans and plain radiographs fail to reveal additional myelomatous involvement of the pelvic ring. When consulted regarding weight-bearing recommendations you advise:

A

WBAT

Pathologic fractures of the pelvis that do not involve the acetabulum are generally treated non-surgically. Patients may bear weight as tolerated. Aggressive analgesic management may be required. Mechanical insufficiency of the acetabulum can only be managed surgically with reconstruction using screws or pins with protrusioacetabular component.

155
Q

Which serotype of botulinum toxin is used clinically?

A

Botulinum toxin B

The pharmacologic properties of the serotypes differ, and only serotypes A & B are available for clinical use. Botulinum toxin type A1 (onabotulinumtoxinA marketed as Botox/Vistabel by Allergan Inc. (Irvine, CA), abobotulinumtoxinA (ABO) marketed as Dysport/Azzalure by Ipsen (Paris, France), incobotulinumtoxinA (INCO) marketed as Xeomin/Bocouture by Merz Pharmaceutical Gmbh (Frankfurt, Germany)) is the most common form of botulinum toxin used. Botulinum toxin type B1 is also available (rimabotulinumtoxinB marketed as Myobloc/Neurobloc by US WorldMeds in USA and by Eisai Europe Limited in Europe).

156
Q

Which of the following conditions increases in incidence with the number of years following a traumatic spinal cord injury?

A

Pressure Injury

In a multicenter analysis of medical complications following traumatic spinal cord injury (TSCI), the development of pressure sores was the most common complication. The frequency of pressure ulcers increases over time from 15% at year one post-TSCI to 23% at ten years post-TSCI. The incidence of pulmonary embolus and pneumonia both remained unchanged during this ten-year period. Despite this, pulmonary complications remain the leading cause of death following TSCI, accounting for as much as 21% of deaths after the 1st year post-TSCI. The incidence of atelectasis and pneumonia were found to be higher in two distinct populations at years 1, 2, 5, and 10 post-injury: those with complete tetraplegia and those over the age of 60. The incidence for DVT peaks within the first 2 weeks of injury, and individuals remain at increased risk for the first 12 weeks.

157
Q

A 25-year-old runner reports having constant right shin pain of 3 weeks duration. She is currently running 40 miles/week and is training for her second marathon this year. X-ray of her right leg reveals no fracture line while her MRI shows bone marrow edema on T1 and T2 weighted images plus periosteal edema with no clear fracture line. Which of the following is the most appropriate recommendation for future activities?

A

6-8 weeks nonimpact activities

Runners with constant shin pain may have developed either medial tibial stress syndrome (MTSS or “shin splints”) or tibial bone stress injury (TBSI or “stress fracture”) due to a variety of factors related to overtraining , hyperpronation, low tibial bone density, etc. The Fredericson MRI criteria for MTSS and TBIS has been used to guide treatment. There are four grades (I-IV) with the patient in this question falling under Grade III. She would require 6-8 weeks of nonimpact activities.

158
Q

How much pronation can occur in the residual limb of a wrist disarticulation?

A

A patient with a wrist disarticulation can commonly perform up to 120o of pronation/supination.

159
Q

What is the leading cause of cardiac-related death in younger athletes?

A

Idiopathic hypertrophic subaortic stenosis

Coronary artery disease is the most common cause in the older athlete, and idiopathic hypertrophic subaortic stenosis is the most common cause in younger athletes.

160
Q

In patients with rotator cuff tendinitis, sleeping with a pillow between the affected arm and the trunk decreases tension on which tendon?

A

Supraspinatus

With the arm in a slightly abducted position, there is less tension on the supraspinatus tendon, and this will help prevent compromise of blood flow in the watershed area of the tendon.

161
Q

Muscles in the anterior compartment of the leg

A

The muscles involved in foot eversion are within the lateral and anterior compartments.

Anterior compartment: The anterior compartment contains the:
- tibialis anterior
- fibularis tertius
-extensor hallucis longus
- extensor digitorum longus.
- Deep fibular nerve
- atnerior tibial vessels

Lateral Compartment:
- Fibularis longus and brevis
- Superficial fibular nerve

Of these four muscles, fibularis tertius and extensor digitorum longus act as foot evertors. Collectively, all four are innervated by the deep fibular nerve and contribute to ankle dorsiflexion.

Deep Posterior:
- Tibialis posterior
- FHL
- FDL
- Popliteus
- Tibial nerve

Superficial Posterior Compartment:
- Gastrocs
- Plantaris
- Soleus
- Sural Nerve

162
Q

The strongest restraint to valgus stress in the elbow is the:

A

Anterior bundle of the medial collateral ligament

The medial (ulnar) collateral ligament (also known as the Tommy John ligament due to rupture in throwing athletes) is the primary restraint due to valgus force. Specifically, the anterior oblique band is loose in flexion and taut in extension.

163
Q

Which of the following provide sensory innervation to the posterior sacroiliac joint?

A

L5 Dorsal Ramus

The majority of the posterior sacroiliac joint is believed to be innervated by the L5 dorsal ramus and S1, S2, S3 lateral branches. No medial branches are involved in the innervation of the sacroiliac joint.

164
Q

What is the strongest predictor of a seizure-related motor vehicle crash?

A

Duration of seizure free interval

Seizures are responsible for 1% of all motor vehicle collisions. Minimum requirements for length of seizure-free time before a person can drive vary from state to state but range from 3-12 months. The strongest literature confirmed predictor of a seizure related motor vehicle collision is duration of seizure free interval. A 12-month seizure free interval prevents 80% of crashes. A 3-month interval prevents 50% of crashes. Other risk factors for motor vehicle collisions in individuals with seizures include age, gender, generalized or complex seizures, partial complex seizures, history of multiple seizures, and noncompliance with antiepileptic medications. The probability of a seizure related collision is decreased by a long seizure free period, reliable aura, and the use of the least sedating, but most effective, medication.

165
Q

Risk factors for entrapment neuropathies in musicians include:

A

Risk factors for entrapment neuropathy include improper hand position, small hand size, joint laxity, and changes in the performer’s practice schedule.

166
Q

According the American Disabilities Act of 1990, a person with a disability must typically have access through the employer to which of the following?

A

Worker’s compensation

The American with Disabilities Act of 1990, amended 2008, states that a covered entity shall not discriminate against a qualified individual with a disability. This applies to job application procedures, hiring, advancement and discharge of employees, worker’s compensation, job training and other terms, conditions and privileges of employment. This gives similar protections against discrimination to individuals with disability as those stipulated in the Civil Rights Act of 1964, which made discrimination based on race, religion, sex, national origin and other characteristics illegal. Benefits, such as insurance or transportation to work, need to be consistent with the company’s policies for all employees and in accordance with state law. Public transportation does need to accommodate people with disabilities.

167
Q

A musician presents with insidious onset of dorsal wrist pain that is worse with use of the hand and wrist. You diagnose her with intersection syndrome, which occurs when the abductor pollicis longus and extensor pollicis brevis cross over which muscles:

A

ECRL and ECRB

Intersection syndrome involves the second dorsal compartment of the wrist. It occurs where the abductor pollicis longus and extensor pollicis brevis cross over the extensor carpi radialis longus and brevis. Pain and swelling occur 4-8cm proximal to the wrist. Crepitus may be palpable with flexion and extension of the wrist.

168
Q

Which carpal bone fracture is most associated with causing an ulnar neuropathy at the wrist?

A

The carpal bone fracture most associated with causing an ulnar neuropathy at the wrist is a hamate fracture as it is located on the ulnar aspect of the wrist at the same location where the ulnar nerve travels. The scaphoid and lunate bones are located on the radial aspect of the wrist and therefore a fracture will most likely not affect the ulnar nerve. Capitate fractures are especially rare because the capitate bone sits in a protected position in the hand.

169
Q

The usual time of onset of diabetes insipidus in patients with traumatic brain injury is:

A

Diabetes insipidus after TBI usually has an onset 10-days after trauma when the antidiuretic hormone (ADH) stored in the posterior pituitary is depleted.

170
Q

Cardiovascular Complications of OSA include: Cardiopulmonary complications of obstructive sleep apnea include

A

right ventricular failure, ventricular hypertrophy, pulmonary and systemic hypertension and alveolar hypoventilation.

171
Q

The most frequently dislocated joint in the hand is the:

A

The proximal interphalangeal joint may dislocate in volar, dorsal, or lateral directions when an axial load is applied to the hyperextended joint. Occasionally an avulsion fracture at the base of the middle phalange occurs at the volar plate attachment.

172
Q

The most common risk factor for congenital limb deficiency is:

A

Vascular condition

Congenital limb deficiencies occur during the 3rd to 8th weeks of gestation. It is estimated that 34% are vascular, 32% have no known cause, 30% are genetic, and 4% are caused by a teratogenic agent.

173
Q

Components of the posterolateral corner of the knee

A

The components of the posterolateral corner of the knee are: lateral collateral ligament, popliteus tendon, posterolateral joint capsule, bicep femoris tendon, peroneal nerve, lateral head of the gastrocnemius, lateral meniscus and posterior meniscofemoral ligament. Any of these structures may be injured in a severe sprain of the knee

174
Q

The most common cause of cervical radiculopathy is foraminal narrowing due to:

A

The most common cause of cervical radiculopathy is foraminal narrowing due to facet or neurocentral joint hypertrophy. Nucleus pulposus herniation occurs less frequently in the cervical spine compared to the lumbar spine, and is the cause of pain in only 20% to 25% of cases. Degenerative spondylolisthesis also typically occurs in the lumbar spine and is relatively rare at other spinal levels. When it occurs in the cervical spine, it is usually secondary to arthritis in the facet joints. Synovial chondromatosis in the cervical facet joint is rare.

175
Q

What is the most common endocrine abnormality following a traumatic brain injury?

A

Growth hormone deficiency has been identified as the most common hormonal abnormality after traumatic brain injury (TBI). Growth hormone is the most common hormone deficiency after TBI, followed by adrenocorticotropic hormone, gonadotropins (FSH and LH), and thyroid stimulating hormone

176
Q

Collapse after running marathon most likely due to

A

Postural hypotension

177
Q

Which type of nerve fibers is responsible for vibration perception?

A

Large, myelinated sensory fibers

Vibration, proprioception, light touch, and deep tendon reflexes are all dependent on large, mye-linated sensory fibers. Small, myelinated and unmyelinated sensory fibers are responsible for pin prick and pain sensation, cold perception, and light touch.

178
Q

The clinical practice of avoiding local corticosteroid injection into the Achilles’ tendon because of the risk of tendon rupture is based upon:

A

Case reports and case series

179
Q

Where is the ground reaction force vector located during mid-stance?

A

In mid stance, the ground reaction vector lies anterior to the ankle, anterior to or through the knee axis, and posterior to the hip center. The passive torques created by this vector alignment are ankle dorsiflexion, knee extension and hip extension.

180
Q

Which organ system is responsible for the leading cause for rehospitalization for medical complications after a traumatic spinal cord injury?

A

Overall, diseases of the genitourinary system such as urinary tract infection are the most common reason for rehospitalization, followed by diseases of the skin (pressure ulcers), then respiratory and musculoskeletal systems. Respiratory diseases were more likely associated with hospital admission in persons with tetraplegia, while diseases of the skin were more likely associated with rehospitalization in those with paraplegia.

181
Q

The most common etiology for ischemic stroke in children is:

A

Arteriopathy is the most common cause of arterial ischemic stroke in children. Cardiac disease (congenital or acquired) and prothrombotic conditions (such as Factor V Leiden) are also causes of ischemic stroke in children. Arteriovenous malformations are a common cause of hemorrhagic (not ischemic) stroke in children

182
Q

During the second rocker of stance phase, or ankle rocker phase:

A

During the second ankle rocker the tibia advances over the ankle-foot complex from approximately 10 degrees of plantar flexion at the end of loading response to 10 degrees of dorsiflexion at the end of midstance. The gastroc-soleus contracts eccentrically to control the speed of deceleration of forward tibial progression. During this phase there is maximal pronation at the foot. Options B & D are incorrect; they describe the first rocker (heel rocker) in the loading response. The first rocker begins at initial contact and ends when the foot-flat position is achieved in loading response. During this rocker a controlled deceleration of the foot (plantar flexion) occurs under the control of the eccentrically contracting tibialis anterior. The foot also pronates with flattening of the medial arch, and hind foot in valgus to allow shock absorption. Option C is incorrect as it describes the third rocker (toe rocker). The third rocker begins as the heel rises off the ground surface and body weight rolls over the first metatarsophalangeal joint through push off in terminal stance. Here the foot moves into supination and the heel into varus. The tibialis posterior locks the midfoot so the foot can act as a rigid lever. Concentric contraction of the gastroc-soleus and flexor hallucis longus provide push off force.

183
Q

Which of the following causes of traumatic spinal cord injury has been steadily increasing over the last 3 decades?

A

Falls

Motor vehicle crashes remain the most common cause of traumatic spinal cord injury (TSCI) in the United States, and account for 38.6% of the injuries. Falls remain the second most common cause of TSCI. The incidence for DVT peaks within the first 2 weeks of injury, and individuals remain at increased risk for the first 12 weeks 2.2%). Violence, primarily from gunshot wounds, remain the 3rd leading cause of TSCI (14%). Sports-related TSCI are currently the 4th leading cause of injury (7.8%).

184
Q

Which of the following distinguishes the ASYMMETRIC tonic neck reflex (ATNR) from the SYMMETRIC tonic neck reflex (STNR)?

A

The ATNR is also known as the fencer position and is a neonatal reflex that disappears by 6 months of age. It is never obligatory in normal children. The STNR appears about 6 months and disappears by 1 year, and it provides postural stability as the child makes the transition from crawling to standing.

185
Q

Botulinum toxin injections should not be performed when a patient is taking which of the following, due to potentiation of the botulinum toxin?

A

Aminoglycoside antibiotics (gentamyin) can potentiate the effect of botulinum toxins and botulinum toxins should not be utilized for patients taking this class of antibiotic. Other medications that can potentiate the effect of botulinum toxins include quinidine, anticholinergics, and curare-like compounds.

186
Q

What is the most commonly affected joint in patients with juvenile idiopathic arthritis (JIA)?

A

The knee is the most commonly affected joint in JIA. The wrist and elbow are commonly involved. Hip involvement is a poor prognostic indicator. The cervical spine and temporomandibular joint are more commonly involved in children than adults.

187
Q

Borders and contents of the carpal tunnel

A

The borders of the carpal tunnel are made up of an anterior border which is the transverse carpal ligament which runs from the hamate and pisiform medially to the scaphoid and trapezium laterally. The posterior border of the tunnel is made up of the carpal bones. The contents of the carpal tunnel are the median nerve which provides motor innovation to the first and second lumbricals and the thenar muscles except the deep portion of the flexor pollicis brevis and sensory innovation to the thenar eminence, the palmer aspect of the first to third digits and the lateral half of the fourth digit. The tendons that run inside the carpal tunnel are the flexor digitorum superficialis, the flexor digitorum profundus and the flexor pollicis longus.

188
Q

Total lung capacity can be defined as

Tidal volume is defined as

Residual volume

Forced fital capacity

A

Tidal volume is the amount of gas moved in resting inspiratory effort

TLC is the amount of gas within the lungs at the end of maximal inspiration

RV is the amount of gas within the lungs at the end of maximal expiration

Forced vital capacity is the amount of air that is forcibly exhaled following maximal inspiration

189
Q

Infants and children with birth brachial plexus palsy are most at risk for subluxation of the shoulder in which direction?

A

Glenoid dysplasia with posterior shoulder subluxation is frequently a complication of children after birth brachial plexus palsy. This is related to asymmetric weakness of shoulder external rotators and abductors relative to shoulder internal rotators and adductors

190
Q

In the frontal plane, movements and stabilization are controlled primarily by:

A

Quadratus Lumborum

The obliques are responsible for flexion and rotation of the trunk and spine. The lumbar paraspinals have limited actions as spine stabilizers due to an extremely small lever arm, and instead control spinal flexion via eccentric activation. The quadratus lumborum acts as a spine stabilizer and when activated this leads to spinal extension.

191
Q

Which organized sport has the highest incidence of spine and head injuries in the United States?

A

American Football

Football has the highest incidence of spine and head injuries, followed by boxing, gymnastics, ice hockey, and wrestling.

192
Q

In a two-week-old baby, abnormality of which primitive reflex aids in diagnosis of Erb’s palsy?

A

Moro reflex shows shoulder abduction and elbow flexion initially with sudden neck extension. Asymmetry of this reflex signals a neurologic abnormality. It is valuable in assessing those active movements for patients with upper plexus involvement. The grasp reflex (a) is not affected in upper plexus lesion. The Babinski reflex can be normal in infants up to age 12 months. Parachute (d) reflex does not appear until at least age 5 months.

193
Q

structures is hyaline cartilage the predominant cartilage type?

A

Cartilage is a type of connective tissue found throughout the body, made up of cells called chondrocytes, which produce and maintain an extracellular matrix consisting of collagen and proteoglycans. There are three types of cartilage in the body - hyaline cartilage, fibrocartilage, and elastic cartilage. Hyaline cartilage is the most common type and is found in articular surfaces, such as the knee joint. Fibrocartilage is the strongest type of cartilage and is found in structures such as intervertebral discs and menisci of the knee. Elastic cartilage is more flexible and found in structures such as the external ear, larynx, and epiglottis.

194
Q

To reduce the risk for neural tube defects such as spina bifida, daily folic acid supplementation:

A

Is recommended for all women of childbearing age

Since up to 50% of pregnancies are unplanned, folic acid supplementation is recommended in all women that are of child-bearing age. Folic acid supplementation should begin prior to when a woman learns she is pregnant because neural tube closure occurs during the first four weeks of gestation. Women designated as high risk (personal or family history of neural tube defect, seizure medications, etc) should be given 10x dosing of folic acid (4-8mg/day). Since dietary folate is increased by fortified grains, folate deficiency is associate with a low-carbohydrate diet. Folate fortification is not regulated for grains designated as organic.

195
Q

A 16-year-old gymnast presents with shin pain, disordered eating, and amenorrhea. The diagnosis of stress fracture is confirmed via bone scan. Your pharmacologic treatment recommendation would consist of:

A

Treatment for the female athlete triad should include a multidisciplinary team approach, consisting of a gynecologist or primary care physician, dietitian, and psychologist. Pharmacologic treatment consists of 1000-1300mg of calcium daily, and 600-800 IU of vitamin D with higher doses if found to be clinically deficient. The use of oral contraceptive pills and hormonal replacement has not been demonstrated to improve bone mineral density or reduce stress fractures. Bisphosphonates have limited evidence as a basis for efficacy, and carry the risk of teratogenicity.

196
Q

What is the most common type of voiding dysfunction among elderly individuals?

A

Overactive bladder, also referred to as detrusor overactivity, is the most common type of voiding dysfunction among incontinent elderly individuals. Outlet obstruction (caused by prostatic hypertrophy) is the 2nd most common cause of incontinence among elderly male individuals, while stress incontinence is the second most common cause of incontinence among elderly females.

197
Q

The most common site of maximum tenderness with a navicular stress fracture is:

A

Dorsally between the tibialis anterior and EHL tendons

198
Q

The amplitude of the endplate potential (EPP) above the threshold value needed to generate a muscle fiber action potential is called a(n):

A

SAFETY FACTOR

199
Q

A 16-year-old gymnast with persistent back pain is diagnosed with spondylolysis. This patient is expected to have a defect in which part of the spinal anatomy?

A

A stress fracture of the pars interarticularis is known as spondylolysis. The pars interarticularis is the part of the lamina between the superior and inferior articular processes.

200
Q

Cardiovascular and pulmonary adaptations noted with aerobic training:

A

increased stroke volume and peak cardiac output - increased respiratory muscle strength, maximal voluntary ventilation - reduced dyspnea

201
Q

The validity of a functional outcome measurement tool is defined as the ability to the tool to measure:

A

What it is designed to measure

The validity of a functional outcome measurement tool is defined as the ability of the tool to measure what it is designed to measure. The ability to measure different outcomes simultaneously does not impact the validity of the instrument, but the validity of the tool would need to be established for each of the outcomes being measured. The ability of two different raters to obtain the same conclusion is referred to as inter-rater reliability. Freedom from random error is also related to the reliability of the instrument.

202
Q

Which of the following cancer drugs is LEAST neurotoxic?
A. Bortezomib
B. Cyclophosphamide
C. Paclitaxel
D. Lenalidomide

A

Cyclophosphamide is not generally neurotoxic. Bortezomib, paclitaxel, and lenolidomide are all neurotoxic.

203
Q

In which plane are the thoracic zygapophyseal joints aligned?

A

The thoracic zygapophyseal – or facet – joints are aligned in the coronal (frontal) plane. The cervical facet joints are aligned in the coronal oblique plane. The lumbar facet joints are aligned in the sagittal oblique plane.

204
Q

nerves is known to refer pain towards the head?

A

Dorsal Ramus of C3

The C1-C3 spinal nerves are primarily responsible for transmission of pain signals from the neck to the head.

Specifically, the ventral ramus of the C1 spinal nerve innervates the short muscles of the suboccipital triangle and the atlanto-occipital joint;

the C1 recurrent meningeal branch joins C2 and C3 to innervate the medial atlantoaxial joint and the dura mater of the upper cervical spinal cord;

the C1, C2, and C3 sinuvertebral branches innervate the dura mater over the clivus in the posterior cranial fossa,

the ventral ramus of the C2 spinal nerve innervates the sternocleidomastoid and the trapezius as well as the lateral atlantoaxial joint; the dorsal ramus of C2 innervates the splenius capitis and semispinalis capitis and its medial branch innervates the occipital area (greater occipital nerve);

the C3 ventral ramus innervates the prevertebral musculature; the C3 dorsal ramus (lesser occipital nerve) innervates the posterior cervical musculature, splenius capitis, cervicis, longissimus capitis, semispinalis cervicis, and multifidus,

and its superficial medial branch innervates the C2-3 zygapophyseal joint (third occipital nerve);

and the C3 sinuvertebral branch innervates the C2-3 intervertebral disc.

Trigeminocervical nucleus is a region of the upper cervical spinal cord where sensory nerve fibers in the descending tract of the trigeminal nerve (trigeminal nucleus caudalis) are believed to interact with sensory fibers from the upper cervical roots. The functional convergence of sensorimotor fibers in the spinal accessory nerve (cranial nerve 11) and upper cervical nerve roots converge with the trigeminal sensory descending tracts, allowing the bidirectional referral of painful sensations between the neck and trigeminal sensory receptive fields of the face and head.

205
Q

What is the anterior boundary of the rotator interval?

A

The rotator interval is bordered anteriorly by the superior aspect of the subscapularis tendon and posteriorly by the anterior aspect of supraspinatous tendon.

Contents of the rotator interval include the capsule, superior glenohumeral ligament, coracohumeral ligament, and long head of the biceps tendon.

The rotator interval is best viewed with the patient in modified Crass position which allows for inspection of the intracapsular biceps tendon. The biceps tendon will appear hyperechoic on ultrasound with a hypoechoic separation from the supraspinatus.

206
Q

45-year-old obese female complains of insiduous onset of pain in the medial ankle and hindfoot, between medial malleolus and navicular tuberosity. Which of the following pathologies is likely the cause of her pain?

A

Posterior tibialis tendon dysfunction cause the pain in the medial hindfoot and medial ankle (between medial malleolus and navicular tuberosity). It is the most common cause of acquired pes planus. Typically, there is no history of significant trauma. With progression, the patient may complain of pain in the lateral hindfoot/midfoot due to impingement of calcaneus on fibular and calcaneocuboidal joint and overloading.

Peroneus brevis tendon dysfunction and calcaneocuboid arthritis typically cause pain in the lateral hindfoot. Cuboid subluxation cause lateral midfoot pain with instailbity in later part of stance phase of gait.

207
Q

In talipes equinovarus (clubfoot), which bone is the main source of abnormality?

A

The talus is often small and externally rotated, creating abnormalities of the talocalcaneonavicular joint. The other bones listed are impacted in club foot; however are not the main source of deformity.

208
Q

Which nerve root is most commonly affected post-operatively in cervical decompression?

A

The C5 nerve root is the most frequently involved root after cervical decompression. Surgery. The involvement of other cervical nerves (C6, C7, or C8) has also been reported. Among the proposed mechanisms for the development of C5 palsy, the most acceptable etiologies have been reperfusion injury of the spinal cord after decompression of a chronic compressive lesion of the cervical cord or the tethering effect of the nerve root secondary to posterior drift of the spinal cord after cervical decompression

209
Q

Which of these has the strongest correlation with functional outcome after stroke?

A

Severity of stroke is most strongly correlated with outcomes and is reflected well in the baseline NIHSS scoring system. Motor improvement in the first few days after early treatment is also a strong correlate of good outcome. The scale includes dysarthria and extinction/inattention (neglect) but not dysphagia.

210
Q

A 5-year-old male sustains a traumatic spinal cord injury resulting in tetraplegia. Which complication is most likely?

A

Greater than 90% of children with spinal cord injury prior to reaching skeletal maturity develop scoliosis. Venous thromboembolism is rare in pre-pubertal children with SCI as is hypercalcemia. Heterotopic ossification rates are low in children.

211
Q

Following a right hip fracture due to a fall, a healthy elderly patient was on bed rest for a week prior to undergoing intramedullary pinning of his fracture, and being allowed to weight bear as tolerated on his right leg. At this point, the effects of the inactivity on his system include a decline in:

A

During bed rest, there is reduced hydrostatic pressure and decreased antidiuretic hormone secretion, resulting in increased diuresis, which leads to a maximum decline in blood volume in the first 1 to 2 weeks of inactivity. Plasma volume decreases to a greater extent than red cell mass, resulting in increased blood viscosity. Bed rest also causes increased bone resorption and increase in urinary calcium excretion above normal levels within 2 weeks.

212
Q

the diagnostic criteria for cervicogenic headaches as defined by the International Headache Society?

A

The IHS diagnostic criteria of cervicogenic headache includes the following: pain referred from a source in the neck, clinical, laboratory, or imaging evidence of a disorder or lesion within the cervical spine or soft tissues which could be a valid cause of headache, evidence that the pain can be attributed to said lesion (which can include either demonstration of clinical signs implicating the pain source as the neck or abolition of the headache after diagnostic blockage of cervical structures), and pain that resolves within 3 months of successful treatment of the causative lesion. Further, “abolition of headache” is defined as > 90% reduction in pain.

213
Q

risk factors for developing venous thromboembolic disease in the setting of brain tumor is:

A

Malignancy itself is a risk factor for thromboembolic disease. The majority of cases that develop in patients with brain tumors occur post-operatively. Risk factors include larger tumors, supratentorial location, presence of intraluminal thrombosis in the tumor pathologic specimen, age older than 60 years, presence of hemiparesis, and use of chemotherapy.

214
Q

A child with spinal muscular atrophy is admitted to the hospital with pneumonia due to aspiration and has copious secretions on physical examination. What is the best diagnostic test to assess his secretions?

A

Fiberoptic endoscopic evaluation of swallow

Bedside evaluation of impaired swallow includes assessment of oral sensation, oral reflexes, postural abnormalities, motor assessment of face, lips, tongue, palate and lar-ynx, level of arousal, ability to follow directions and management of saliva. Bedside screening tools useful to physicians include the 3-oz water swallow test or the Toronto Bedside Swallowing screening test (TOR-BSST)

Modified barium swallow (MBS) or videofluoroscopic swallowing study (VFSS) evalu-ates swallowing by radiographically visualizing swallow using various consistencies of barium. It provides real-time viewing of oral, pharyngeal and cervical esophageal phases of swallowing.

Fiberoptic endoscopic evaluation of swallowing (FEES) directly visualizes laryngeal and pharyngeal structures involved in swallowing. It is portable, lacks radiation, visual-izes secretions, and detects structural abnormalities of the pharynx and larynx. It does not assess the oral stage of swallowing or the cervical esophagus.

Manometry determines the pressures and relative timing of pharyngeal contraction and relaxation of the upper esophageal sphincter during swallowing.

215
Q

the best single predictor of falling in the elderly based on gait characteristics?

A

Stride-to-stride variability

Gait in the elderly is characterized by increased double-limb support, as well as by slower speed, shorter stride length, and a broader base of support. Pelvic rotation decreases, and postural responses are slower. Gait studies have been done to identify factors associated with falling in the elderly. In the subset of older people who fall, stride-to-stride variability in gait is increased. Studies have showed increased stride-to-stride variability in length, speed, and double support was associated with actual falling. Stride-to-stride variability was the best single predictor of falling.

216
Q

A patient with a traumatic brain injury is classified at a Rancho Los Amigos level III (3). Their stage of recovery can best be described as:

A

Rancho Los Amigos level III corresponds to the minimally conscious state with only local response in the Braintree Neurologic States of Recovery from Diffuse TBI scale. The postconfusional/emerging independence corresponds to level VI/VII. Vegetative state corresponds to level II. The confusional state is at level IV, V and early VI.

217
Q

Most common complication s/p VP shunt?

A

Infection; About 15% of infants with myelomeningocele require VP shunting immediately. The other 85% do not require immediate shunting but need close follow up. Cervical level myelomeningocele is not the most common location with associated hydrocephalus to occur.

218
Q

During a normal walking gait cycle, the portion designated as the loading response:

A

The loading response during stance phase of the gait cycle typically occurs between 2-10% through the cycle, and represents the transition in weight support from the previous stance limb to the current. It starts immediately after initial contact, and ends once the contralateral foot leaves the ground. It occurs during double limb support, which is initiated by initial contact. As cadence increase, loading response (and all components of stance phase) shorten in duration.

219
Q

Which of the following is least likely to control knee hyperextension during stance phase in the presence of genu recurvatum and normal ankle range of motion?
A. KAFO with single axis unlocked knee joint
B. Dual-channel AFO with posterior pin
C. Articulated AFO set in dorsiflexion
D. KAFO with posterior offset knee joint

A

A. KAFO with single axis unlocked knee joint

Single axis knee joint (free knee) permits unrestricted flexion and extension to 180 degrees in the sagittal plane while providing medio-lateral stability.
Option B: A pin in the posterior channel would prevent plantarflexion. This would be useful in foot drop. The effect on the knee would be to create a knee flexion moment, thereby reducing knee hyperextension.
Option C is intended to prevent excessive ankle plantarflexion in stance phase, subsequently reducing knee hyperextension by creating a knee flexion moment.
Option D: In a posterior offset knee joint the hinge is located posterior to the knee joint and ground reaction force; thus, it extends the knee and provides great stability during early stance phase of the gait. This joint flexes the knee freely during swing phase and is contraindicated with knee or hip flexion contracture and ankle plantar flexion stop.

220
Q

What is considered most likely to predict successful return to driving after a stroke?

A

On-road driving evaluation offers more relevant, comprehensive, and objective assessment of fitness to drive than other methods. For this reason, it is considered a criterion standard by many evaluation programs in the assessment of resumption of safe driving in individuals with severe disability. Cost and availability of such programs can be prohibitive, but alternate tests that predict successful performance on road evaluations, though more practical, such as use of Trail Making A and B, Rey-Osterreith Complex Figure Design, and evaluation of road knowledge and reaction time may not be as accurately predictive. Medical evaluation is still needed to assess contraindications posed by seizure disorders, TIA risk, poorly controlled diabetes, or cardiac conditions associated with unpredictable lapses of consciousness. Patients with aphasia should receive reasonable accommodations for participating in written driving exams if this is required.

221
Q

In an inpatient rehabilitation setting, greater functional gains are typically seen in which individuals with which diagnosis?
A. TBI
B. Ischemic Stroke
C. Primary Brain Tumor
D. Metastatic Brain Tumor

A

A. TBI

Several studies have shown significant functional improvements from admission to discharge as measured by Functional Independence Measure (FIM) scores. Overall FIM change, FIM efficiency, and discharge to home rates were comparable between primary and metastatic tumor groups. Patients admitted to inpatient rehabilitation facilities with a diagnosis of traumatic brain injury tend to have greater total FIM change compared to patients with brain tumors. Patients admitted to inpatient rehabilitation facilities with a diagnosis of stroke appear to have equivocal total FIM change compared to patients with brain tumors. However, one study noted a greater FIM efficiency in the tumor cohorts. Researchers have found that patients who received chemotherapy or radiation during inpatient rehabilitation did not have a significant difference in length of stay (LOS) or FIM gains when compared to those who did not.

222
Q

Brain tumors in which lobes are the LEAST likely to be associated with seizures?

A

Occipital

Cortical tumors have a higher incidence of associated epilepsy than noncortical deeper lesions. Tumors involving the frontal, temporal, and parietal lobes are more commonly associated with seizures than occipital lesions.

223
Q

Excess angles of greater than 15 degrees are not recommended in wheelchair seating systems because they:

A

Flatten the lumbar curve

Increasing the seat angle (degree of seat dump) too much closes the hip angle and forces the pelvis to rotate backwards, flexing the spine, and flattening the lumbar curve, which adds pressure and shear forces at the sacrum, increasing the risk of skin breakdown. Option A is incorrect as when seated in a WC with a typical seat angle of 5 degrees, the total body weight is transferred to the seat, backrest leg and arm rests of the WC. Increasing the dump somewhat means that more of the weight is transferred to the spine & trunk, and presses against the wheelchair back providing more stability when seating. Too much dump displaces the weight from the trunk and back to the ischial tuberosities. Patients with higher level spinal disabilities gain security and safety with the use of seat dump. Also, manual riders are able to exert more push with less effort through their arms and shoulders. Option B is incorrect as too much seat dump increases knee flexion (small knee flexion angle), which transfers the weight of the trunk over a small area at the ischial tuberosities. Option C is incorrect as closing the hip angle too much increases hydrostatic pressure, decreasing venous return in the lower extremities.

224
Q

Which of the following has been shown to decrease the risk of development of post-thoracotomy pain following chest wall surgery?

A

Epidural catheter placement with continuous epidural analgesia has been shown to decrease the incidence of post-thoracotomy pain. The other listed choices have not been studied in regard to the incidence and development of post-thoracotomy pain.

225
Q

Activity limitations and/or participation restrictions in an individual with a health condition, disorder, or disease is called:

disadvantages related to disability that result in reduced participation in normal life roles.

A significant deviation, loss, or loss of use of any body structure or body function in an individual with a health condition, disorder, or disease is called:

A

Disability

Handicap

Impairment

226
Q

The process in which an inpatient rehabilitation unit receives information about outcomes as compared to centers in the geographic region is referred to as:

A

Benchmarking

227
Q

What is the most common mechanism of hydrocephalus in patients admitted to inpatient rehabilitation after traumatic brain injury?

A

Decreased reabsorption of CSF

228
Q

The most useful single modality for the diagnosis of osteomyelitis in a pressure ulcer?

A

The histologic examination of bone biopsy specimens is the gold standard for the diagnosis of osteomyelitis. X-rays are most cost efficient but give false negative or false positive results for osteomyelitis in 50% of cases. Swab cultures of the wound have poor correlations with the organisms infecting the bone.

229
Q

On the videofluoroscopic swallow study of a patient with right hemiplegia after a stroke, pharyngeal weakness with significant pharyngeal residue was noted on the right side. As a swallow strategy for this patient, the speech therapist recommends:

A

Turning the head to the right side

Tucking the chin to the chest is typically recommended in patients with laryngeal penetration or aspiration to enhance airway protection. For unilateral pharyngeal weakness with significant pharyngeal residue, turning the head to the weak side directs the bolus towards the stronger side and reduces upper esophageal sphincter pressure. Tilting the head to the strong side can also help to redirect the bolus to the stronger side.

230
Q

Frailty can be described as:

A

Frailty can be defined as age- and disease-related loss of adaptation, such that events of previously minor stress result in disproportionately biomedical and social consequences. Frailty is difficult to quantify. There is a generalized decline in multiple systems with a loss of functional reserve.

231
Q

An individual presents to your office with a Stage II sacral pressure ulcer, which appears clean, with no necrotic tissue or fibrinous exudate, and only a slight amount of serosanguinous drainage. In order to optimize wound healing, you suggest:

A

An occlusive dressing

An occlusive dressing will help to maintain a moist environment, which is ideal for wound healing. Careful monitoring should occur to prevent excessive moisture that can lead to tissue maceration that prolongs healing time. Allowing eschar to form will inhibit healing and prevent accurate staging of the ulcer. Wound healing is optimized only when a clean wound base exists. Wet to dry dressing changes are used only when mechanical debridement is required; in this case, there is no necrotic tissue or fibrinous debris. Vacuum-assisted closure is usually used on stage III and stage IV wounds.

232
Q

Criteria to define frailty include

A

Weakness with decreased grip strength

Slowness on 15 ft walk test

Unintentional weight loss ≥5% over past year

Three or more of the following five criteria must be met for the diagnosis of frailty:
Weight loss of ≥5% in last year or Body mass index (BMI) less than 18.5 or unintentional weight loss of more than 10 pounds in the past year.
Exhaustion. The Center for Epidemiologic Studies Depression Scale is used.
Weakness (decreased grip strength measured by a dynamometer)
Slow walking speed of greater than6 to 7 seconds for 15 feet, or scoring less than the 20th percentile, stratified for sex and height
Decreased physical activity (males <383 kilocalories, kcals); females <270 kcals) or complete inactivity.
The stages of frailty are:
0 criteria are present: Non-frail stage
1– 2 criteria present: Prefrail stage
3– 5 criteria present: Frail stage